88
KAPLAN LSAT PREP LSAT RELEASED TEST XVII EXPLAINED A Guide to the December, 1995 LSAT KAPLAN The answer to the test question.

Prep Test 17

Embed Size (px)

Citation preview

KAPLAN LSAT PREP

LSAT

RELEASED TEST XVIIEXPLAINED

A Guide to the December, 1995 LSAT

KAPLANThe answer to the test question.

1996 Stanley H. Kaplan Educational Center Ltd

All rights reserved. No part of this book may be reproduced in any form, byphotostat, microfilm, xerography or any other means, or incorporated into anyinformation retrieval system, electronic or mechanical, without the written permissionof Stanley H. Kaplan Educational Center Ltd.

© K A P L A N 1

SECTION I:

LOGIC GAMES

LSAT PREP ______________________________________________________________ LSAT Test XVII Explained: Section I

2 © K A P L A N

GAME 1 — Seven Appointments(Q. 1-5)

The Action: We’re asked to order the appointments for seven patients—P, Q, R, S, T, U,and W. These appointments have been chronologically numbered, 1 through 7, by the testmakers. This should give you a big clue as to what your setup should look like. The KeyIssues will be:

1) What patient has what appointment?2) What patient’s appointment can, must, or cannot be consecutive with what otherpatient’s appointment?

The Initial Setup: Keep this simple. The appointments are numbered 1 to 7, so theintuitive way to represent this is, almost certainly, to write the numbers 1 through 7 acrossthe page from left to right. Remember to keep a roster of the patients somewhere:

P Q R S T U W

1 2 3 4 5 6 7

The Rules:

1) Recognize that this rule does not mean that Q must have the appointment immediatelybefore W (although she could). This just says that Q’s appointment must come sometimebefore W’s. Be sure you get the order right. Shorthand this by drawing a Q, a few dots tothe right of it, and then a W.

2) Second rule, same as the first. Again, be sure you have the correct order. Draw a U, afew dots following it, and then a P.

3) Build this directly into your sketch. Either R or T will always have appointmentnumber 3. Under the 3 in your sketch, draw “R or T.”

4) At first this rule may have looked complex, but all it’s saying is that R and S must haveconsecutive appointments. The specific order of their appointments is up for grabs,however. Draw “RS or SR” to give yourself a reminder.

Key Deductions: There’s not a lot more to deduce, because the rules have little incommon. Just be sure that you understand the workings of each rule. In a game asstraightforward as this, if you mess up a rule, you can lose quite a few easy points.

LSAT PREP _______________________________________________________________ LSAT Test XVII Explained: Section I

© K A P L A N 3

It’s always a good idea to try to identify the most important entity or entities—the ones thatpromise the most help. Normally, that’s the one or two entities mentioned multiple timesin the rules. However, in games like this one, where no entity is mentioned twice, youshould also pay special attention to the most concrete rules. Here, only R and T are tied toan appointment. Rule 3 told us that either R or T must go in slot 3. This is a big hint that Rand T are going to be the most important entities. In other words, throughout the game youwill be dealing with whether R or T will be the one to go in appointment 3, so thatpromises to be a good jumping-off-point and head start in the questions to follow.

The Final Visualization: Here’s what you have as you go on to the questions:

P Q R S T U W

1 2 3 4 5 6 7 R or T

Q . . . W U . . . P SR or RS

The Big Picture:

• When a game gives you a clue about the setup (as it does here with “chronologicallynumbered 1 through 7” in the opening paragraph), use it. The game is announcingits action and its type.

• Never rush through your work on the opening paragraph and rules. You need aproper foundation in order to make any game work. If you reversed the order ineither of the first two rules, you’re sunk. You know that the testmakers figure onsome examinees' mixing them up. Why should that be you?

• Be sure to take a few seconds, before you jump into the questions, to review the rulesand ask yourself: What is likely to be most significant?—Which entities and rules arethe most helpful or concrete? Which of the entities or rules should I appeal to first,in any given question?

LSAT PREP ______________________________________________________________ LSAT Test XVII Explained: Section I

4 © K A P L A N

The Questions:

1. (E)Use this acceptability question to test your grasp of the game’s inner workings. Follow theKaplan technique and compare each rule against each choice, eliminating all the choicesthat violate a rule. (A) has P before U, which is the opposite of Rule 2. (B) has someoneother than R or T in appointment 3. That’s a no-no according to Rule 3. (C) separates R andS, which we know is not allowed according to Rule 4. (D) has W before Q, which isverboten according to Rule 1. We’re left with our answer, choice (E).

• Acceptability questions are extremely helpful. Besides offering a quick'n'easy point,they offer a golden opportunity to practice working with the rules and entities. Bythe end of such a question, you are given one situation that works within the game’sparameters (the correct answer), and you’re also given four situations that violate therules (the wrong answer choices)...all of which will help you better understand therest of the game.

2. (E)The stem puts W in appointment 2 and P in appointment 5. It also tells you that you’relooking for the choice that must be true, so the four wrong answers either could be true orcannot be true. What rule has W in it? Rule 1 says that Q has an earlier appointment than W.The only appointment before W, who takes appointment 2, is 1. Which rule contains P?Rule 2 says that U must precede P. P is in 5, Rule 3 tells us that either R or T is in 3, so theonly place for U is in appointment 4. Scan the choices. Is “Q in 1” or “U in 4” a choice?Sure it is. U is in 4, choice (E).

If you didn't stop to check the choices at that point, you probably continued on, andrecognized that assigning R to appointment 3 won't fly—that would separate R from S(Rule 4). So T will take appointment 3, leaving R and S to slots 6 and 7, in either order. Nowthat everything's done, it's clear that (E) must be true and that (B) and (D) are both false,while (A) and (C) are only possibly true.

• Whenever you’ve deduced something new, stop and scan the choices to see if yournew deduction is an answer choice. Often (as is the case here) you can take thedeductions further, but it’s a waste of time to do more work than is needed to get theanswer.

• If you had reversed the order of the entities in Rule 1 or Rule 2, this is the questionwhere you might have picked up on your mistake. If this happens on test day, don’twaste time beating yourself up about it. Be grateful for catching your mistake in timeto do something about it.

LSAT PREP _______________________________________________________________ LSAT Test XVII Explained: Section I

© K A P L A N 5

3. (B)Like question 2, in this “must be true” question, the four wrong answer choices eithercould be true or absolutely must be false. Don’t just stare at this stem, your eyes glazingover; get the new information down in your test booklet. Take your time and be sure youget the order right. You should have written “TUR.” In what rules have you seen theseentities before? They’re in plenty of them; let’s start with Rule 4. S has to be next to R, andsince U is on one side of R, S must be on the other. Now we have a group of 4 entities,“TURS.”

Where else are these entities mentioned? Rule 3 told us that either T or R must takeappointment 3. Both R and T are in the group of four. This has “limited option” written allover it. Whenever you can narrow things down to two or three possibilities, it’s almostalways worth it to go ahead and work them out. With R is 3: T will have to takeappointment 1, U in 2, R in 3, and S in 4. We’re left with P, Q, and W. As long as Q is beforeW (as Rule 1 mandates), these three will fill out the 5, 6, 7 slots. “Option 1,” as it were, lookslike this:

1 2 3 4 5 6 7T U R S (P) (Q . . . W)

For the second possibility, T in slot 3, just move the “TURS” foursome over and get T in 3,U in 4, R in 5, and S in 6. Go back to the rules. Rule 2 says that P must follow U, and theonly appointment left after U is 7. Q and W are left to take appointments 1 and 2. Rule 1says that Q goes before W, so Q gets appointment 1 and W appointment 2. Everything is setin this “Option 2.” Here’s what it looks like:

1 2 3 4 5 6 7Q W T U R S P

With both of these options fully flushed out, it should be a simple (and quick) matter ofchecking each choice.

(A) In the first option, P could be in 5, Q in 6, and W in 7. (A) doesn’t have to be true and isnot the answer.

(B) In the first option, S is in 4 and P must be in 5, 6, or 7. In the second option, S is in 6 andP is in 7. S must precede P. Either way, choice (B) is true, and on test day you'd choose itand move on. For the record, we’ll go through why the remaining choices needn’t be true.

(C) In the second option, we see that not only is S not immediately before Q, S is actuallyafter Q.

(D) Stay in the second option, where W (in appointment 2) doesn't immediately precede P(which takes appointment 7), and you see why (D) needn't be true.

(E) Back to the first option; S is in 4 and W is in 6 or 7 (Q must be before it, so W can’t be in5). W needn’t precede S.

LSAT PREP ______________________________________________________________ LSAT Test XVII Explained: Section I

6 © K A P L A N

• When you can narrow down the number of possibilities to three or fewer, it usuallypays to go ahead and work them out. The time you spend will be more than made upfor when you can blow through the choices.

• It’s never a waste of time to recopy a master sketch. Indeed, recopying is an excellent,time-efficient way to set up a clean framework for a question AND get a rules brush-up besides. However, while recopying a sketch is no trouble at all, rethinking it fromscratch is deadly. And so, to facilitate recopying:

• ...It's crucial that you keep your master sketch neat and untouched. It represents thebasic workings of the game before any extra information has been added, and youneed to be able to consult it throughout the game.

4. (B)In this “could be true EXCEPT” question, four of the answer choices either could be trueor must be true; the choice we’re looking for will be the one that is false. Depending on howyou felt your time management was going, you may have decided to skip this question onyour first go-around. Since four choices will work within the games setup, you canprobably count on spending a lot of time here.

The question stem tells us that P immediately precedes S. Where else have we seen P andS? Rule 4 demands that R be next to S. P is on one side of S, so R must be on the other. Rule2 says that U must be before P (maybe right before P, maybe not). So that gives us“U . . . PSR.”

We’re used to deciding whether R or T goes in the 3 slot, so let’s look there next. Can R goin 3? Nope. At least U, P, and S are before R. No way to put R in the 3 slot, so T must gothere. Now, where can we put the “U . . . PSR” block? PSR are all together, so they can’ttake the 1, 2 slots. These three will have to follow T in the 4, 5, 6 or 5, 6, 7 slots. U can goanywhere as long as it precedes P. Q and W will fill in the leftovers as long as Q precedesW as Rule 1 requires. Now let’s check out the choices.

(A) The ordering U, Q, T, P, S, R, W works and shows that R can immediately precede W.Since (A) can be true, it's not the answer.

(B) If T immediately precedes Q, then T is in 3, Q is in 4, and our PSR block must occupy5, 6, and 7, respectively. But Rule 1 says that W comes after Q, and we have no slots leftafter Q. (B) is impossible and is the answer. Quickly, here are the orderings that show thatthe remaining choices all could be true.(C) and (E) are both true in: Q, W, T, U, P, S, R.

(D) is true in: U, Q, T, W, P, S, R.

• Learn to ask yourself the important questions, such as: “In what rules have I seenthis entity?,” and “Where does this new information lead me now?”

LSAT PREP _______________________________________________________________ LSAT Test XVII Explained: Section I

© K A P L A N 7

• Earlier, we discussed identifying the aspects of a game that promise to be the mostimportant. Notice that in every question so far, you’ve had to place either R or T inappointment 3. By now, doing so should be second nature. After you’veincorporated any new information into a question, you should immediately askyourself “O.K., now what about R and T? Which one takes slot 3 this time?”

5. (D)Another “must be true” question, so the four wrong answer choices will be things thatcould be true or must be false. From the stem we get (make sure you get the order right)“PTW.” In what rules have we seen these entities? Rules 1 and 2 mean that Q and U must gosomewhere before our “PTW” block. Now, as usual, who will take slot 3, R or T? Here wehave Q, U, and P who must all precede T. T can’t be in 3, so R must take that appointment.The “PTW” block must follow R in either 4, 5, 6 or 5, 6, 7. But remember Rule 4. S must takethe 2 or 4 slot in order to be next to R. Q and U have to precede “PTW,” so S, Q, and U willfill in the 1, 2, 4 slots. The only possibility for our “PTW” block is in slots 5, 6, and 7.

(A) R is in 3 and W is in 7. W comes after R, not before.

(B) U could take slot 4. U needn’t precede R.

(C) U in 2 and S in 4 fulfills all the requirements and shows that S does not have to comebefore U.

(D) R is in 3 and P is in 5. Indeed, R must precede P. (D) must be true and is the answer.

(E) S in 2 and Q in 4 shows that (E) doesn’t have to be true.

• Don’t rush through complex or unusual question stems. Take the time to workcarefully with the information and get it down correctly on paper. From there, testthe new information against the rules and you’ll find the answer. If you don't takeyour time and don’t proceed with care, what’s the point?—you won't end up with aright answer anyhow, and all you'll have achieved is wasted time and frustration.

• Checking each answer choice is always an option available to you, but activelypursuing an answer is almost always the better option, because it’s quicker and moredirect. Even here where you finally had to check the choices, we managed to greatlyreduce the time required through our solid deductive work on the new information.

LSAT PREP ______________________________________________________________ LSAT Test XVII Explained: Section I

8 © K A P L A N

GAME 2 — Employee Committees(Q. 6-12)

The Action: We’re called on to distribute six employees among three committees—Policy,Quality, and Sales. Each committee has three employees per committee for a total of nine.The six employees are divided up into officers—F, G, and H—and supervisors—k, l, and m(we’ll use CAPS for the officers and lowercase for the supervisors to keep them separate).The Key Issues will deal with:

1) What employee is on what committee?2) What employee can, must, or cannot be on the same committee as what other employee?3) What employee can, must, or cannot be on more than one committee?

Key Issue 3) comes about because we have six employees filing nine committee slots. Thisis the first thing you had to recognize: There are more slots to fill than people to fill ‘em, sosome employees must sit on more than one committee.

The Initial Setup: Keep this setup very simple. Get the committees down and the numberof employees in each. That and making a note of the employees is all that’s needed at thisstage:

OFFICERS supervisorsF G H k l m

Policy Quality Sales

___ ___ ___ ___ ___ ___ ___ ___ ___The Rules:

1) Each of the three committees must contain at least one of our officers. Make a note ofthis under each committee in your sketch.

2) Basically a loophole-closer but very necessary. No employee will be left out of thegame.

3) Huh? You probably blinked on this one. Can it possibly be that helpful? Yep: All threeofficers fill out the Policy committee, so go ahead and write them in. 1/3 of the game’swork is over. Just be sure that you remember that the officers aren’t done: They can still goon other committees (and, in fact, we need at least one officer per committee, as per Rule 1).4) G, an officer, can’t be on the same committee as l, a supervisor. Write “No Gl” as areminder. Saying it to yourself (silently) a couple of times may cement it. Wherever weplace G, l must go elsewhere, and vice versa.

LSAT PREP _______________________________________________________________ LSAT Test XVII Explained: Section I

© K A P L A N 9

5) A good, solid, concrete rule; we can go ahead and write k into the Sales committee.Remind yourself that k can still go on the Quality committee if need be.

Key Deductions: What originally looked like a grouping game with three groups, turnsout to be a game with only two groups: The Policy committee is filled. The main thing tokeep in mind is the flexibility in this game. Just because one of the employees is assigned toa committee, doesn’t mean that she or he can’t join an additional committee.

You may or may not have picked up on one additional bit of deductive thinking that goeslike this: Every entity must join at least one committee (thanks to Rule 2). The only entitiesnot placed in a committee by the rules are l and m, and of course, as supervisors, they areclosed out of the Policy committee. Of the three slots in the Sales committee, k takes oneand an officer takes another so, at most, one slot in Sales is available for l and m.Meanwhile, on the Quality committee, one slot is occupied by an officer; therefore, at mosttwo slots are available for l and m. When all is said and done: Where are l and m serving?There are only three possibilities. l in Quality, m in Sales; m in Quality, l in Sales; or both land m in Quality. In all of these possibilities, we see that either l or m (or both) must jointhe Quality committee. So we can enter “l or m” over one of our Quality committee spaces:At least one of that pair must be in the Quality committee. This is tricky thinking, but itproves very useful with question 10 in particular, and throughout the game.

The Final Visualization: Here’s what we have going into the questions:

OFFICERS supervisorsF G H k l m

Policy Quality Sales

_F_ _G_ _H_ ___ _l/m_ ___ ___ _k_ ___ Off. Off. Off.

NEVER G l

LSAT PREP ______________________________________________________________ LSAT Test XVII Explained: Section I

10 © K A P L A N

The Big Picture:

• Pay special attention to the numbers aspect of every game. The minimums andmaximums required in a game are always important—especially in games, like thisone, where the entities can occupy more than one spot.

• Always move from what you’re told or you realize CANNOT be true, to what MUSTbe true. Don’t wallow in negativity! Drive the information to what is certain. That’swhat games work is all about.

The Questions:

6. (D)The word “acceptability” isn’t used in this stem, but it’s an acceptability question all thesame. The phrase “can be assigned together” is the big tipoff. We’re dealing with the SalesCommittee, so take what we know about that committee and look at each choice. Rule 5told us that k must go on the Sales committee, so axe (A) and (B), which don’t include her.Rule 4 prohibits G and l from being on the same committee, and down goes (C). Rule 1requires at least one officer on each committee, so (E), which tries to get away with threesupervisors, can be crossed off. We’re left with (D), the answer.

• We’ve said it before: Got to love those acceptability questions. One of the few nicethings the test makers ever did was to make acceptability questions a standard partof games.

7. (C)In a “must be true” question, the four wrong choices either can be true or must be false.We’re given two pieces of information, so start with the one that’s more concrete, and hereit’s that H is in only one committee. H is already on the Policy committee, so that’s it forH—it’ll be left to G and F to take care of the other two committees. The second piece of infois that F and m must be separated. m has to be somewhere, so whatever committee m is on,the only officer left to join m is G. Rule 4 says that G and l can’t be on the same committee,so l will have to be on the other committee with F.

Who can fill in the remaining slot in each committee? H is out of the running, according tothe stem. F can’t join the Gm committee (the stem says no Fm) and neither can l (Rule 4,again). The only employee who can join Gm is k. What about the Fl committee? Again, thestem rules out m (no Fm), and Rule 4 says no G. Again, only k can join Fl. The Policycommittee is F, G, H (as always). We don’t know exactly which threesome is on which ofthe two remaining committees, but one will be G, m, k, and the other will be F, l, k. We seethat (C) must be true, k is on exactly two committees.

(A) No. G, m, k could be on the Sales committee.

(B) No, F, l, k could be on the Quality committee.

LSAT PREP _______________________________________________________________ LSAT Test XVII Explained: Section I

© K A P L A N 11

(D) No, l is on exactly one committee, either Sales or Quality.

(E) No, m is on exactly one committee, either Sales or Quality.

• Don’t worry if you can’t figure out everything in a question. Here, for instance, youdon’t know which group was in the Sales committee and which was in the Qualitycommittee. You don’t have to. When you’ve deduced something and hit a lull, go tothe answer choices.

• When you receive two pieces of information in a question stem, deal with them oneat a time, starting with the one that’s most concrete.

8. (B)In this “CANNOT be true” question, we’re looking for the choice that must be false. Thefour wrong choices either must be true or can be true. In questions like this with no newinformation, it’s worth the time to scan the answer choices, keeping the rules in mind. (B)and (C) should spring out as likely candidates, since assigning anyone to all 3 committeesradically restricts the other assignments—perhaps fatally. Can G or H sit on all threecommittees? What rules mention G or H? Rule 1 mentions H, but only to included H onthe Policy committee. G, on the other hand, is part of Rule 1 and Rule 4. Rule 4 says that Gand l must be separated—and there you have it. Every employee (l included) must sit on atleast one committee. Since l and G can’t sit on the same committee, whatever committee l ison cannot also include G. G cannot sit on all three committees; (B) must be false and is theanswer. Here are the committee groups that show that each wrong choice could be true:

(A) Policy: F, G, H; Quality: G, m, k; Sales: H, l, k

(C) Policy: F, G, H; Quality: H, m, k; Sales: H, l, k

(D) and (E) Policy: F, G, H; Quality: F, l, m; Sales: F, l, k

• The “active” way into a question—trying to deduce and pre-phrase an answerdirectly—is usually the fastest. But if you can’t see one, don’t despair, because youcan always try out the choices hands-on. Don’t be leery of trial and error. Generally,if you try, you won’t err.

• Get in the habit of scanning the choices in advance of working on the question.Doing so lets you recognize exactly what’s being asked, and will give you a clue as tohow much work will be enough, which in turn saves valuable time. Why do morework, and spend more time, than necessary?

LSAT PREP ______________________________________________________________ LSAT Test XVII Explained: Section I

12 © K A P L A N

9. (E)Another “must be true” question; the four wrong choices either could be true or must befalse. Another stem with two pieces of information. Starting with the more concrete, F sitson three committees, so beyond just Policy, F also sits on Quality, and on Sales with k. Asfor G sitting on two committees, that would be Policy (as always) plus either Quality orSales—leave it at that for the moment. Recognize that l and m are the only ones not yetassigned to a committee. Rule 4 dictates that whatever committee G sits on, l must sit onthe other. So we’re down to two “limited options.” If G is on Quality, then l must fill outSales with F and k. That leaves m to fill out Quality with G and F. Option 1 looks like this:Policy: F, G, H; Quality: F, G, m; Sales: F, k, l

Alternatively, we could put G on Sales. l must go on Quality, and m is left to fill outQuality. Option 2 looks like this:Policy: F, G, H; Quality: F, l, m; Sales: F, G, k

Now, just check the choices against these two options, and we see that no matter what, msits on the Quality committee. (E) must be true and is the answer.

(A) In the second option, we see that G needn’t be on the Quality committee.

(B) In the first option, we see that G needn’t be on the Sales committee.

(C) Actually in both options, k can only be on the Sales committee.

(D) In the second option, we see that l needn’t be on the Sales committee.

• When faced with two or three “limited options,” work them out in your test booklet.Why try to figure them out in your head, when you can quickly work them out infront of you where they can be easily referenced?

LSAT PREP _______________________________________________________________ LSAT Test XVII Explained: Section I

© K A P L A N 13

10. (E)Another “acceptability” question; this time we’re looking for an acceptable Qualitycommittee. Here’s where the extra thinking up front pays off. We’ve already deduced thateither l or m must be on the Quality committee. (If you don’t remember how, look back atKey Deductions.) The right answer must include either l or m or both, so right away crossoff (A), (B), and (C). None of them include the needed l or m. From there treat this like anormal “acceptability” question. Compare each rule against each choice. (D) tries to put Gand l together which is a violation of Rule 4. (E) is all that’s left and is the answer.

• Even if you didn’t make the “l or m in Quality” deduction, you could have stilleliminated (A), (B), and (C) just by keeping in mind the numbers aspect of the game.We know that all of the employees must be used, and l and m are the only “floaters”in this game. (A), (B), and (C) are all guilty of not leaving enough space for l and m tosit on a committee. You could have tested each choice, but it’s quicker to axe them allin one fell swoop.

11. (E)Another “must be true” question, and again the four wrong choices either can be true ormust be false. The stem says that l is on two committees. The only two open to l are Qualityand Sales (Policy is full of officers). Rule 4 rules out G from being the officer in either ofthese committees, so either F or H will fill those slots. k is still on Sales, so that committee isfull with l, k, and either F or H. Meanwhile, m has yet to be assigned a committee, so itmust be Quality. Quality is now full with l, m, and either F or H. Scan the choices. We seethat m must sit on the Quality committee, and (E) is the answer.

(A) It could be H that is on the Sales committee.

(B) Absolutely not. Since l in on Sales, no way that G could be too.

(C) It could be F that is on the Quality committee.

(D) Nope. k is on Sales. The Quality committee is F or H, l, and m.

• Don’t let the testmakers throw you with vague hints. Turn what’s vague intosomething definite. They tell you that “l is assigned to exactly two committees.” Fine;but it’s your job to figure out which two. Take command!

LSAT PREP ______________________________________________________________ LSAT Test XVII Explained: Section I

14 © K A P L A N

12. (B)Like question 8, in this “CANNOT be true” question, the four wrong answers either can betrue or must be true. To prove a choice wrong, all we have to do is find an instance thatshows that the choice can be true. Here’s where your previous work comes in handy.Review the previous questions. If a situation could be true then, it can be true now andmay be crossed off.

(A) Question 11 showed that the following grouping was possible: Policy: F, G, H;Quality: H, l, m; Sales: H, l, k. In this setup, F and G sat on exactly one committee. It couldbe true then, it can be true now. Cross off (A).

(B) If F and H are on exactly one committee, then that committee must be Policy (Rule 3has already placed them there). Rule 1 says that each committee must have at least oneofficer, and the only officer left is G. G goes on both Quality and Sales. But l must join atleast one of these two committees (everyone must be used), yet Rule 4 prohibits putting Gand l together. Answer choice (B) cannot be true and is the answer. For the record, here arewhy the remaining choices don’t measure up:

(C) Back to Question 11 where we saw the following grouping: Policy: F, G, H; Quality: F,l, m; Sales: F, l, k. There we saw G and H sitting on exactly one committee. It could be truethen, it can be true now. Eliminate (C).

(D) The following grouping shows that F and M can be on the Sales committee: Policy: F,G, H; Quality: H, l, k; Sales: F, k, m.

(E) The following grouping shows that G and K can be on the Quality committee: Policy:F, G, H; Quality: G, k, m; Policy: F, k, l.

• Use your previous work whenever possible. Of course, that’s a natural argument forleaving your various sketches untouched. If you redraw your sketch as needed, allof your work will be there whenever it’s needed. Don’t erase anything but yourerrors (if any).

LSAT PREP _______________________________________________________________ LSAT Test XVII Explained: Section I

© K A P L A N 15

GAME 3 — A Day’s Meals(Q. 13-17)

The Action: In this straightforward, eerily familiar game (take a look at “Seasonal Sports”from PrepTest XIV, section 1, Game 4 if you need more practice on EXACTLY this kind ofgame), we’re called on to match up two people, Vladimir and Wendy, with the food—fish,hot cakes, macaroni, poached eggs, or omelet—that each eats during one day’s four meals:breakfast, lunch, dinner, and snack. The Key Issues will, predictably, be:

1) What foods can Vladimir and Wendy eat for each meal?2) What foods can’t Vladimir and Wendy eat for each meal?

The Initial Setup: Set up a little schedule:

Brkfst Lunch Dinner Snack

Vlad

Wendy

The Rules:

Start with the most concrete rule. You’ll find it way down there at the bottom.

7) For lunch, Wendy eats an omelet. Great, build it right into the schedule.

1) This rule is huge because it greatly restricts the possibilities in this game. For each meal,Vladimir and Wendy will eat different foods. This is why it’s best to start with the mostconcrete rule. We just placed a big “O” for omelet in Wendy’s lunch slot with Rule 7 (whatWendy, no hamburger?). Now we can put rule 1 into effect. Vladimir cannot have anomelet for his lunch food. He must choose between fish, hot cakes, and macaroni. As far asthe rest of the game goes, this is a rule that will have to be uppermost in yourconsciousness, along with:

LSAT PREP ______________________________________________________________ LSAT Test XVII Explained: Section I

16 © K A P L A N

2) Not only do Wendy and Vladimir not duplicate each other’s food in a meal—the betterto share and sample, I guess—but neither person eats the same thing twice. Again—nothingmuch to draw here; it’s a fact that has to stay alive in your brain throughout.

3) The only foods that Vladimir and Wendy can eat for breakfast are hot cakes, poachedeggs, and omelet. Note that since Wendy eats an omelet for lunch, Rule 2 means that shecan’t eat an omelet for breakfast. Enter her remaining possibilities into the schedule.

4) The only foods available for lunch are fish, hot cakes, macaroni, and omelet. We knowWendy eats an omelet here, so Rule 1 prohibits Vladimir from having an omelet as well.List his remaining possibilities on the schedule.

5) For dinner, the two must choose among fish, hot cakes, macaroni, and omelet. Rule 2 isstill in effect—Wendy has had her omelet for the day—so list each person’s remainingpossibilities.

6) Only fish and omelet may be eaten for the snack. This is big. Having gotten an omelet forlunch, Wendy is left to have fish here. Rule 1 demands that Vladimir have somethingdifferent from Wendy, so Vladimir’s snack is omelet. Enter both facts on the schedule.

Key Deductions: By dealing with the most concrete rule (Rule 7) first, we were able to domost of our deducing as we went along. We’ve done some excellent thinking and shouldbe prepared to blow through the questions. Before hitting the questions though, you canand should narrow things down a bit further. We deduced both snack orders: Wendy eatsfish and Vladimir eats an omelet. Remember Rule 2: Wendy eats fish for her snack, so fish isunavailable for her throughout the rest of the day. Vladimir eats an omelet, so that’s out forhim. Omelet is already out for Wendy since she eats one for her lunch. This means thatomelet can be crossed off for both people throughout the day.

The Final Visualization: Here is our extremely useful master sketch:

Brkfst Lunch Dinner Snack

Vlad H/P F, H, M F, H, M O

Wendy H/P O H/M F

LSAT PREP _______________________________________________________________ LSAT Test XVII Explained: Section I

© K A P L A N 17

The Big Picture:

• Don’t be intimidated when there’s a large number of rules. They may look scary onthe page, but actually, a large number of rules normally makes for a much moremanageable game. Some games with fewer rules turn out to be more difficultbecause they’re much more ambiguous and abstract—they tell you less, so they helpyou less.

• What type of sketch you use is, of course, up to you. Just always be sure that yourscratchwork is accurate and neat. Its purpose, remember, is to remind you of therules quickly and accurately. Sloppiness, or sloppy thinking, undermines bothgoals.

• Many games hinge on the rules that CANNOT effectively be drawn. Here, noscratchwork is going to help you remember that Vladimir and Wendy never have thesame food at the same meal (Rule 1), and that neither eats the same food twice (Rule2). You either understand those facts and carry them with you, or you fumble, crash,and burn. This is one big reason why the Kaplan methods—which stress thinking,and emphasize scratchwork as an aid to thinking—are so superior to those who tellyou that in games, “drawing is all.”

The Questions:

13. (E)A “must be true” question with no new information is almost always testing a bigdeduction. We made the extra deductions that Wendy eats fish for her snack and Vladimireats an omelet for his (look above if you need a reminder of the work). The test makerschose the former as choice (E).

• If you didn’t make any big deductions upfront, seeing this “must be true” questionwith no new information is a big clue that you need to go back and look at the rulesand setup closer.

14. (D)What must Vladimir eat? Again, this question is most likely testing our big deductionabout Vladimir. The only thing we know for certain about his daily diet is that he eats anomelet for his snack, choice (D).

• It should be very clear to you how important it is to take the time upfront to thinkthrough all possible deductions. We’ve just been able to blow through the first twoquestions in around a half a minute total.

LSAT PREP ______________________________________________________________ LSAT Test XVII Explained: Section I

18 © K A P L A N

15. (D)In this “could be true” question, the four wrong choices must be false. This questionbenefits from a quick redrawing of the master sketch. Looking at the sketch, we see that theonly meal during which Wendy can eat macaroni is dinner. Write an “M” in her dinnersquare. Rule 1 means that macaroni is out for Vladimir’s dinner. The only other meal whereVlad can eat the needed macaroni is lunch. Write an “M” in his lunch square. Back to Rule2 which dictates that macaroni is now off-limits for both people in all of the other meals.Note the possibilities and here’s our new sketch:

Brkfst Lunch Dinner Snack

Vlad H/P M F/H O

Wendy H/P O M F

Now, it’s a simple matter to check each choice against our new sketch and find the one thatcan be true. We see that Wendy could indeed eat hot cakes for breakfast. Choice (D) couldbe true and is the credited answer.

(A), (B), and (C) Vladimir eats macaroni for lunch, only.

(E) Wendy eats macaroni for dinner.

• Don’t hesitate to redraw your master sketch when needed. It is crucial that you keepthe original untouched. Messy work leads to messy thinking which leads to missedpoints.

LSAT PREP _______________________________________________________________ LSAT Test XVII Explained: Section I

© K A P L A N 19

16. (B)Another “could be true” question, so the four wrong choices must be false. Follow thischain of reasoning carefully: If Wendy doesn’t eat macaroni, then the only food left for herdinner is hot cakes. Since she eats hot cakes for dinner, she can’t eat them for her breakfast.The only food left for her breakfast is poached eggs. Wendy eats poached eggs forbreakfast, so Vlad can’t eat them. The only food left for Vlad’s breakfast is hot cakes. Vladeats hot cakes for breakfast, so he can’t eat them for any other meal. Here’s what thisincarnation of the master sketch looks like:

Brkfst Lunch Dinner Snack

Vlad H F/M F/M O

Wendy P O H F

Once again, it’s a simple matter to check the choices and find the one that could be true.Vladimir could eat fish for lunch, and (B) is the answer.

(A) and (D) No, Vladimir eats hot cakes for breakfast; Wendy eats the poached eggs.

(C) No, since Vladimir eats hot cakes for breakfast, the only foods open to him for lunchare fish and macaroni.

(E) No, Wendy eats hot cakes for dinner.

• Again, don’t be afraid to redraw your sketch. The worst thing you can do is to sit andstare at a question because you don’t think it’s worth your time to redraw.

LSAT PREP ______________________________________________________________ LSAT Test XVII Explained: Section I

20 © K A P L A N

17. (B)In a “cannot be true” question, the four wrong answers are either possibly true, ordefinitely so. Wendy eats poached eggs for breakfast, so Vladimir can’t eat them. The onlyother option for his breakfast is hot cakes. Now hot cakes are unavailable for the rest of hisday. Scan the choices and (B) should leap out at you. We just said that Vladimir has hotcakes for breakfast. Rule 2 is still alive and strong, so he can’t eat hot cakes at any othermeal. (B) cannot be true and is the answer. All of the other choices are quite possible in thissituation.

• This game is a virtual carbon copy of the “Seasonal Sports” game from PrepTest XIV.This is a MAJOR argument for using past PrepTests along with Kaplan’sexplanations to help prepare for your exam. The test takers who knew “SeasonalSports” backward and forward reported crushing this game when it appeared ontheir test. One student said that he had to stifle a laugh when he turned the page andsaw Wendy and Vladimir. Guess which of the four games he did first?

LSAT PREP _______________________________________________________________ LSAT Test XVII Explained: Section I

© K A P L A N 21

GAME 4 — Two Relay Teams(Q. 18-24)

The Action: In this game, you need to distribute eight people—Jack, Karen, Laura, Mark,Nick, Owen, Peggy, and Ruth—among two four-person teams—team X and team Y. Thereis an added twist. Once the people are assigned to a team, the four members of each teamare placed into four legs of the race. We’re also told that the teams run their legs at the sametime. The Key Issues of this grouping/sequencing hybrid are:

1) What people are on what team?2) What people run what leg on their team?3) What people can, must, or cannot be on the same team as what other people?4) What people can, must, or cannot run consecutive legs with what other people?5) What people can, must, or cannot run an earlier leg than what other people6) What people can, must, or cannot run the same leg as what other people?

The Initial Setup: We have two teams with four legs per team. What would be the naturalway to set this up? A simple way would be to write “X” and “Y” for the teams with fourdashes under each for the legs. Remember to list the people off to the side:

J K L M N O P R

Team X Team Y

___ ___ ___ ___ ___ ___ ___ ___ 1st 2nd 3rd 4th 1st 2nd 3rd 4th

The Rules:

A concrete rule sits in the middle of the pack:

4) Mark and Nick are on team Y. Great; build them right into your master sketch under Y,but remember that you don’t yet know which leg each runs on team Y.

1) Jack will be on the same team as Karen. You can jot down “Always J K” as a reminder, orwait one rule and you’ll be able to deduce something concrete.

2) Karen will be on one team and Nick will be on the other. From Rule 4 we know thatNick is on team Y, so Karen must be on team X. Rule 1 just told us that Karen and Jack areon the same team, so put them both on team X.

LSAT PREP ______________________________________________________________ LSAT Test XVII Explained: Section I

22 © K A P L A N

3) Take your time and think this rule through; it’s tricky. Ruth and Peggy don’t necessarilyhave to be on the same team, but Ruth must run an earlier leg than Peggy. (Do a “What If”if you need some further clarification. Imagine that Ruth is on team X and Peggy is on teamY. If Ruth is running the 3rd leg on team X, then Peggy must run the 4th leg on team Y.)Anyhow, write down “R . . . P” to serve as a reminder always to schedule R before P.

5) We can put this rule directly into the master sketch since we know that Jack is on team Xand Mark is on team Y. Under team X’s 3rd leg write “No J,” and under team Y’s 3rd legwrite “No M.”

6) Karen and Laura run the 2nd leg. We know Karen is on team X, and there are only two2nd legs, so if Karen is running the 2nd leg on team X, then Laura must be running the 2ndleg on team Y. Build them both directly into your sketch.

6) Hmmm. We don’t know which team Owen is on, but wherever he is, he’s going to run4th. Write “Owen runs 4th” or something similar to remind you.

Key Deductions: You could spend your time working out countless “What Ifs” about thearrangement of runners and legs. But doing so is pointless. Wait for the questions toprovide more concrete information that will kick off more deduction.

The Final Visualization: Here’s the final master sketch:

J K L M N O P R R . . . POwen runs 4th

Team X Team Y J + 2 M N + 1

___ _K_ ___ ___ ___ _L_ ___ ___ 1st 2nd 3rd 4th 1st 2nd 3rd 4th

No J No M

LSAT PREP _______________________________________________________________ LSAT Test XVII Explained: Section I

© K A P L A N 23

The Big Picture:

• Once again we see the value of starting with the most concrete rule. All of the otherrules usually make much more sense in the light of that one fixed point.

• The setup is the time to think, above all. The drawing is secondary. Consider theimplications of each rule, and if you can build it directly into your master sketchinstead of rewriting it (as in Rule 6 and others), then great, do it.

The Questions:

18. (A)This “must be true” with no new information has complex answer choices. You might havebeen best served by skipping this one your first time around. Remember that the fourwrong choices either could be true or must be false. To eliminate a choice in a “must betrue” question, all you have to do is find an exception. That is, find an instance where withthe given conditions, the stated result isn’t the only one possible.

(A) If Owen is on team X, he must run 4th (Rule 7). Jack can’t run 3rd (Rule 5) and Karen isrunning 2nd (Rule 6). The only leg left for Jack to take is the 1st. Choice (A) must be trueand is the answer. On test day, you would stop here and go on to question 2. For therecord, here are the exceptions that show the remaining choices needn’t be true:

(B) and (E) X: 1st—Jack, 2nd—Karen, 3rd—Peggy, 4th—Owen Y: 1st—Ruth, 2nd—Laura, 3rd—Nick, 4th—Mark

(C) and (D) X: 1st—Ruth, 2nd—Karen, 3rd—Peggy, 4th—Jack Y: 1st—Mark, 2nd—Laura, 3rd—Nick, 4th—Owen.

• Since Owen is assigned a definite leg and Ruth and Peggy aren’t, starting with thechoices that include him, those are answer choices (A) and (D), is smart.

19. (D)Ruth is on team X. Which legs could she run? Well, Rule 6 assigned Karen to the 2nd leg, sono way could Ruth run there. Whoa!: That eliminates answer choices (B), (C), and (D), all ofwhich include the 2nd leg. Answer choices (A) and (D) are left; what is the differencebetween them? They both include 1st, so no need to check and see if she can run there. Theonly question is whether Ruth can run 3rd. Here is a complete setup that shows she can:

X: 1st—Jack, 2nd—Karen, 3rd—Ruth, 4th—PeggyY: 1st—Mark, 2nd—Laura, 3rd—Nick, 4th—Owen.

Both 1st and 3rd are possibilities for Ruth, so (D) is the answer.

• Keep all of the rules in mind at all stages of your work.

LSAT PREP ______________________________________________________________ LSAT Test XVII Explained: Section I

24 © K A P L A N

20. (E)In a “must be true” question, the four wrong choices either could be true or must be false.Owen and Ruth are on the same team. What team? M, N, and L are already on team Y, sothere’s no room for both Owen and Ruth there. They must be on team X. Peggy is left toteam Y.Rule 7 forces Owen to run 4th for team X. Jack can’t run 3rd (Rule 5), so the only leg open toJack is 1st. Ruth is left for the 3rd leg. Scanning the choices, notice that all of the entitiesmentioned are on team Y. We have to take our deductions a little further. Since Ruth runs3rd, Rule 3 forces Peggy to run 4th. Stop right there and scan again. That’s enough, becausewe find Peggy running 4th down in choice (E).

• From time to time, stop and scan the choices. You only want to deduce what’snecessary to answer the question. There are no bonus points for taking thedeductions as far as possible; doing so wastes time that can be better spentelsewhere.

21. (C)In a “can be true EXCEPT” question, the four wrong choices either can be true or must betrue, and the answer must be false. Since there’s no new information, there’s not much to doexcept try out the choices, looking to throw out the possible ones and locate the oneimpossibility.

Consulting your previous work allows you to answer this question more quickly. InQuestion 20, we saw J running 1st and Owen and Peggy running 4th. That could be truethen, so that can be true now: Cross off (A) and (E). In eliminating choices (B) and (E) inQuestion 18, we saw Mark running 4th. He could run 4th then, he can now. Eliminate (B).In that same setup, we also saw Nick and Peggy both running 3rd. They could run 3rdthen, they can now. Axe (D). We’re left with (C), Nick cannot run 1st.

• There’s no need to check to make certain that Nick can’t run 1st. When you’re leftwith one choice, that’s the answer. Keep this new information in mind throughoutthe rest of the questions: Nick can never run 1st.

22. (C)In a “must be true” question, the four wrong choices either could be true or must be false.If Ruth and Peggy are on the same team, what team must that be? M, N, and L are on teamY, so there’s no room for Ruth and Peggy there; they must be on team X. Team X is Jack,Karen, Ruth, and Peggy. Team Y is Laura, Mark, Nick, and Owen. As far as the ordering isconcerned, Owen continues to be a good starting place since he’s set, in the 4th leg. Owenwill run the 4th leg on team Y. Laura is still running 2nd. Mark can’t run 3rd, so only 1st isleft for him. Nick will fill the remaining slot by running 3rd. The complete ordering forteam Y is set:

Y: 1st—M, 2nd—L, 3rd—N, 4th—O.

LSAT PREP _______________________________________________________________ LSAT Test XVII Explained: Section I

© K A P L A N 25

Team X is a little more slippery, but only a little. K is set in the 2nd leg. Jack is forbidden toleg 3, so he will take either leg 1 or 4—and Ruth and Peggy will fill in the remaining slots inthat order (Rule 3); Therefore:

EITHERX: 1st—J, 2nd—K, 3rd—R, 4th—PORX: 1st—R, 2nd—K, 3rd—P, 4th—J.

As correct choice (C) says, Nick runs 3rd. Of the wrong choices, (B) is false—Mark runs 1st;and the others are possible only.

• Keep an eye on your answer choices as you work. As soon as you’ve found a correctanswer—stop!

23. (B)Questions 21 and 23 are identical; and once again we can use our previous work to seekout cases in which choices were true in the past. In the second option for Team X inquestion 22, we saw Jack running 4th and Ruth running 1st. Eliminate (A) and (D). In thefirst option for team X of that same question, we saw Peggy running 4th and Ruth running3rd. Cross off (C) and (D). We’re left with our answer, choice (B). Nick cannot run 4th.

• Use your previous work whenever possible. That’s why it’s very important that youkeep it neat and unerased.

24. (B)This “must be true” question turns out to be pretty easy because we’re so used to dealingwith Peggy and Ruth by now. Peggy is on the same team as Jack. That’s team X. We’re alsotold that she runs 3rd on this team. Karen is running 2nd as always. Where to now? Well,you should be used to dealing with Rule 3. Ruth must run earlier than Peggy. Peggy runs3rd, and both 2nd legs are taken by Karen and Laura. So whichever team Ruth is on, shemust run 1st. Scan the choices and there it is. Ruth runs 1st, choice (B).

• This game, and indeed this very question, dramatically illustrates why it’s soimportant to manage your time well. You don’t want to miss a straightforwardquestion like this at the end of the section, simply because you wasted time on anearlier, tougher one.

26 © K A P L A N

SECTION II:

LOGICAL REASONING

LSAT PREP ______________________________________________________________ LSAT Test XVII Explained: Section II

© K A P L A N 27

1. (B)We’re told that despite the cheapness of cement’s ingredients, its price is influenced byoil’s price, the reason being the high energy needed to turn the ingredients into cement. Theimplication is that oil is needed for that transformation, at least in some cement making,and that’s answer choice (B). Use the Kaplan Denial Test: If choice (B) were false—if nocement kilns used oil—then how would the cost of oil have anything to do with the cost ofcement?

(A) The very use of the Contrast Keyword “Nevertheless” belies this inference. “Oil isnevertheless an influence, despite the cheapness of cement’s ingredients;” in other words, oilis no part of the composition of cement but a part of its cost.

(C) and (D) Both choices go too far. That cement’s price is influenced by oil’s price is amodest claim, one that cannot be read as suggesting a flat-out direct (C) or inverse (D)relationship.

(E) The author implies that at least one factor in cement making—the needed energy—isboth separate from cement’s ingredients and relevant to cement’s cost. So (E) is a highlyunlikely inference.

• That which can be “logically inferred” means that which must be true based on thetext. Accept no substitutes.

2. (E)This is a classic example of the reasoning error sometimes called “circular reasoning.” Theconclusion is signaled by the Keyword phrase “It is clear from this that...,” and notice thatwhat follows is just a rewrite of the previous sentence, “without self-understanding it isimpossible to understand others.”

(A) The flaw described in (A)—mistaking necessity for sufficiency—is an error oftencommitted in LSAT Logical Reasoning arguments, but not in this one. We’re told acondition (self-understanding) that is necessary for a particular result (understandingothers). So answer choice (A) would be correct if the argument went on to conclude that“Anyone who understands himself can understand others.” But that’s not the conclusion, ofcourse; as answer choice (E) recognizes, the conclusion we get is just the evidence restated.

(B) First of all, the first sentence does suggest that there are some people who don’t wantself-understanding, but beyond that, this whole issue of wanting or not wanting self-awareness is irrelevant. The author’s concern is whether one can understand others withoutself-awareness.

(C) No blame is assigned, and even if it were, there’s no indication that the people inquestion “cannot . . . be held responsible” for not understanding themselves or others.

(D) There’s nothing “inherently vague” about the term “self-understanding,” even thoughdifferent people could define the dimension of that understanding in different ways.

LSAT PREP ______________________________________________________________ LSAT Test XVII Explained: Section II

28 © K A P L A N

• Circular reasoning—also known as “begging the question” and “assuming one’sconclusion”—is when the evidence and conclusion are functionally identical, i.e.one is just a rewrite of the other. That’s what happens here. Watch for it in the future.

3. (C)In response to his wife’s desire to sell a painting in order to pay for their daughter’s collegeeducation, the husband offers three different reasons for hanging on to the thing, but it’s thethird reason that relates most to her argument and that is supported by (C). Establishing, as(C) would, that a link to the past supersedes a college education would make thehusband’s argument, and recommendation about the painting, outweigh the wife’s.

(A) This family has owned the painting for some time. (A)’s focus on whether or not a giftshould be “accepted” is totally irrelevant to the issue at hand.

(B) The husband considers the painting a horror, so (B)’s allusion to the work’s “beauty” islikewise irrelevant.

(D) No “promise” such as (D) mentions is ever alluded to by either spouse, and the issue iswhether the painting should be sold, not whether it should be preserved or destroyed.

(E) Au contraire, of course. (E) supports the wife’s position, not that of her husband.

• A “justifying principle” is one that alludes to the key issues in an argument and thataffirms that the conclusion is valid.

• Watch for wrong answers in principle questions to support an opposing point ofview ((E) here) or to veer off into irrelevant issues ((A), (B), and (D) here).

4. (E)This is pretty much a two-for-one deal, since all (E) does is articulate Question 3 (C)’sprinciple in more general terms: The husband sees the family link as an “obligation” that“override[s]” the “practical consideration” of paying for college.

(A) and (D) The husband doesn’t argue that her solution is either impractical or false: Hedoesn’t argue that the painting won’t pay for college, only that that shouldn’t be thegoverning consideration.(B) Since the wife doesn’t say that the painting is lovely (just that it’s valuable), thehusband’s criticism is not a slur on her taste.

(C) If anything, it’s the husband whose recommendation is based on emotion—sentimentalvalue and family connections and all that—not the wife.

• Never assume that when a stimulus generates two questions, the questions mustnecessarily focus on different things. If anything, work on one question generallyhelps on the other.

LSAT PREP ______________________________________________________________ LSAT Test XVII Explained: Section II

© K A P L A N 29

5. (D)The author’s conclusion—that the archaeologists will surely be able to determine theaccuracy of this year’s museum financial report—is based on their universal access to allrelevant documents and, in turn, on the assumption that those documents are going to listall relevant financial transactions. But if they do not, then the reviewers’ determination ofaccuracy may be way off. (D) points out the possibly unwarranted assumption here.

(A) The report examines only each year’s transactions; long-held pieces are irrelevant.

(B) and (C) If the documents under review are complete, then the accuracy of the reportwill be assured; if they’re not, then it won’t. Neither the size nor quality of the collection(B), nor its availability to the public (C), will matter one way or the other.

(E) True, the author doesn’t discuss what the next steps would be if discrepancies werefound between the report and the documents, but he is not logically obligated to do so.The argument is simply that given the documents, the archaeologists will be able todetermine the accuracy of the report—what happens after this is one step beyond theargument.

• Keep your eye on the ball—which in LR means “Keep in mind the question beingasked, and the scope and nature of the conclusion.” Here, the author’s conclusionisn’t about whether or not the report is accurate, but whether or not the reviewerswill be able to make a determination of accuracy. Two very different things!

• Don’t fault an argument for not containing a certain element if that element isn’tnecessary for the logic to work. Just because a choice may be true, like (E), doesn’tnecessarily means it answers the question asked.

6. (D)“Some people argue bla-bla-bla....But this is nonsense.” The sentence demands theresponse: How so? What’s your evidence? This alerts us that that which “is nonsense,” thatwhich demands evidence, is almost certainly the argument’s main point. (D) has it justright: The argument that using the Moon’s helium-3 will solve Earth’s energy problems isnuts because, as sentences 3 and 4 explain, we couldn’t build the reactors that the helium-3would fuel until too late.(A) The feasibility of mining the Moon is taken off the table by the engineer’s “Even if.” Hebypasses that whole question, in order to take up his real issue, which is the feasibility ofusing helium-3 in time to do the world any good.

(B) Current reactors are beside the point. The engineer is concerned with the reactors 50years hence which could, conceivably, be designed to use helium-3.

(C) implies that the people mentioned in line 1 are the engineer’s target, but they’re not; it’stheir point of view that he objects to. Besides, fuels other than helium-3 are nevermentioned, so they could hardly be part of the “main point.”

LSAT PREP ______________________________________________________________ LSAT Test XVII Explained: Section II

30 © K A P L A N

(E) is the gist of the engineer’s last sentence, which is evidence for the main point, not thepoint itself. (If you chose (E), ask yourself: Where is the evidence supporting it? None isprovided, so (E) can’t be the main point the author is trying to make.)

• If no Conclusion Keyword (such as “Therefore”) jumps out at you, read the stimulusand look for a sentence that whispers (or screams) I am the author’s point of view—Idemand evidence. Watch for the other sentences to provide support; the idea they allcome together to support will be your conclusion.

• Remember that the author’s conclusion can appear anywhere—at the beginning,middle, or end. That decision is a matter of taste and style, not of logic.

7. (C)The stem tells us we’ll be looking for a statement that strengthens the argument, so the firststep, as always, is to break down the argument into evidence, conclusion, and assumptions.Smack dab in the middle of the stimulus, highlighted by the always reliable structuralsignal “therefore,” the author tells us the conclusion: “The government should thereforeinstitute a program” to test the toxin level in the fish eaten by the seabirds killed by netfishing. The evidence appears in the other two sentences—The fishing industry currentlyhas no incentive to report the number of seabirds actually killed by net fishing (the firstsentence), and, if the program is adopted, the industry will have a reason to turn in birdsbecause the industry will get desirable information in return (the third sentence). In otherwords, under the program, the author envisions a win-win situation for the governmentand the fishing industry. Since the argument is essentially for the creation of the program, astrengthener must bolster the need for such a program of cooperation. (C) does this nicely:If the government can’t reach its objective (an accurate count of seabirds killed by netfishing) without the cooperation of the fishing industry, a program providing an incentivefor the fishing industry to help out, such as the one outlined, is essential.

(A), if anything, weakens the argument, by working against the fishing industry’s incentiveto cooperate. (A) makes any data that the government gives to the industry less valuable.

(B) is irrelevant; it really doesn’t matter what the government has done in the pastregarding tissue sampling. You might have thought that (B) weakens the argument (similarto (A)), by reasoning that if (B) were true the government might, through inexperience,have difficulty administering the program—However, this reasoning is not supported bythe stimulus. In any case, it’s far from a strengthener.

(D) certainly doesn’t support the idea of the program; the only effect it would have is tohelp destroy the incentive of the fishing industry to participate.

(E) brings up a hypothetical situation, and a result of it, that are totally irrelevant to theprogram itself; the point is not to find out how many seabirds are killed by a reducedamount of net fishing, but to find out how many are killed under current conditions.

• The conclusion, the author’s main argument, can come anywhere in the stimulus,including right in the middle. Don’t assume that it must be contained in the first orlast sentence.

LSAT PREP ______________________________________________________________ LSAT Test XVII Explained: Section II

© K A P L A N 31

• Note the way the structural signals “therefore” and “since” regulate the flow of thelogic in a stimulus like this one. Use them to help you locate the conclusion andunderstand the gist of the author’s argument.

• Recommendations, when they appear, are very often the author’s conclusion. Be onthe lookout for sentences that indicate that a person or group should or must dosomething; chances are that this is the notion the author is ultimately trying tosupport.

8. (B)In this long question stem we’re asked to recognize a situation in which the program is notlikely to achieve its desired result. In other words, we want to undermine the conclusion.The program relies on the fishing industry’s incentive for cooperation. Under the program,the fishing industry must hand over the dead seabirds, which yields the information thatthe government needs—the total number of dead seabirds. The fishing industry, in return,receives valuable information about toxin levels in fish. However, if (B) is true, then thefishing industry can secure the info it wants without turning over all of the dead seabirds,which would stymie the intent of the program to provide an accurate count of seabirdskilled by net fishing.

(A), (C), and (D) All of these choices deal with possible things the government may findonce the seabirds are turned in, and thus all three of these choices discuss things that arebeyond the scope of the argument. We’re looking for something that will affect how manyseabirds are turned in—remember that the government is concerned with getting anaccurate count of the seabirds that died. In order to undermine the government program,we need an answer choice that would make it likely that the number of seabirds turned inwon’t reflect the actual number of seabirds killed by net fishing. None of the factors statedin these choices impacts on that issue at all.

(E), like (A), (C), and (D), has no bearing on the number of seabirds turned in. The onlydifference is that unlike the other wrong choices, the irrelevancy in this one centers on whatthe government may do in a specific situation, rather than what the government may learnabout the nature of contamination.

• Notice the breadth of questioning on the LSAT: From the same situation, you’reasked first to support a plan of action, and then to reverse direction and recognize areason why that plan of action may not achieve its intended result. This is very muchlike the thinking you’ll be required to do in law school, so strive to maintainflexibility in your thought process and to see arguments from all possible angles.This will help you not only on the LSAT, but in your legal studies as well.

• Double question stimuli offer an excellent opportunity for a quick point: Havingthought through the issues involved in order to answer question 7, you’re in a betterposition to tackle what appears to be a more complex question 8.

LSAT PREP ______________________________________________________________ LSAT Test XVII Explained: Section II

32 © K A P L A N

9. (A)The question stem is a fancy way of asking us “so, where’s she going with this?” The firstsentence begins with “some people claim . . .” and the second sentence begins with theword “yet.” Believe it or not, you can pretty much answer the question from thisinformation alone. We know from reading the stem up front that our job is to fill in theconclusion, and the sentence structure of the first two lines should have helped you todeduce it. Think about it: If someone says to you “some people claim X,” and follows thatup with the word “but” or “however” or “yet,” what do you think is going to come next?The speaker thinks NOT X!! Replace X with “elected officials must avoid the appearance ofimpropriety in office,” and NOT X, the conclusion that the word “yet” screams for,becomes choice (A). The rest of the stimulus supports this conclusion. The only possiblereason for X (avoid appearance of impropriety) is Y (maintain public approval andpopularity). No one has an obligation for Y, therefore the conclusion NOT X is confirmed(no elected official has an obligation to avoid the appearance of impropriety).

(B) The author states that the only reason to avoid the appearance of impropriety is tomaintain public approval. The author never states, however, that all elected officials have avested interest in a high public approval rating. The author merely states that if officialswant to maintain public approval they should avoid impropriety. Some politicians couldactually have a vested interest in a low rating. Also, don’t read more into a question thanexists: “vested interest” is not the same as “maintaining public approval,” and “high”public approval rating is not the same as public approval and popularity.

(C) and (E) are way too specific: The author never singles out and discusses “good” electedofficials (C) or “bad” elected officials (E), so any conclusions relating directly to these sub-groups of elected officials is out of place in this argument.

(D) The author states that the only reason an official may ever want to avoid impropriety isto maintain public approval, but this does not mean, as (D) states, that the public neverapproves of an official who appears to have behaved improperly. It’s possible, accordingto the stimulus, that an official can appear to have behaved improperly and still beapproved of by the public.

• Remember, there are two kinds of “conclusion” questions—ones that ask you for theauthor’s stated conclusion, and ones that ask you to recognize the conclusion theauthor is moving towards. Passages attached to the latter type are written to sound asif everything’s there but the last piece of the puzzle. Read such passages withanticipation, as if you’ll be called upon to write the last sentence that ties together theideas of the entire stimulus.

• What a great example to illustrate the power of keywords!! As described above, thisentire passage turns on the “yet” at the end of the second line. Never underestimatethe potential value of such a little word.

• This question clearly illustrates the benefit of reading the question stem first. Inthis stimulus the credited response can be deduced from the first two sentencesalone. If you had read the question stem and understood what you were beingasked to do, you could have chosen the credited response quickly.

LSAT PREP ______________________________________________________________ LSAT Test XVII Explained: Section II

© K A P L A N 33

10. (A)All assumptions, of course, connect evidence and conclusion. The conclusion is heresignaled by “Clearly,” and the evidence basically has two components: (1) The cafeteria’sapples arrive greasy and don’t get washed. (How do you like them apples?) (2) Most fruit issprayed with pesticides that stay dangerous until it’s washed. For (1) and (2) to combineand lead to the conclusion that the cafeteria sells dangerous pesticide-covered fruit, it hasto be true that the apples don’t get a washing somewhere down the road from harvest,which is what (A) is saying. Notice that (A) passes the Kaplan Denial Test: If it’s false—if theapples are washed after harvesting—then there’d be no reason to believe that their greasehas anything to do with the pesticides in the field. The fruit would still be gross, but neednot be dangerous.

(B) Tempting, but this one fails the Denial Test. If (B) is false—suppose that few, if any,pesticides leave a greasy residue—so what? This cafeteria’s apples could be the exceptionto the rule, could still be greasy and dangerous from pesticides. Since denying or negating(B) does no damage to the logic, (B) as is cannot be a necessary assumption.

(C) Patrons’ awareness or lack of same has no impact on whether the apples are, or are not,dangerous and pesticide-covered.

(D) can also be eliminated via the Denial Test. If, contrary to (D), one can wash offpesticides other than those that leave grease, we’d still be no closer to determining thesafety of these particular apples.

(E) Outside the scope: The conclusion is based on evidence about apples, and it holds upjust fine with or without the existence of other greasy fruits.

• You can never be reminded of the Kaplan Denial Test often enough. A necessaryassumption, if denied, must contradict the text. Therefore—follow the contrapositivehere—if an answer choice, when denied, fails to contradict the text, then it’s not anecessary assumption.

11. (E)Did you catch the word “needs” in P’s assertion, and did you then connect it to the conceptof necessity? Q sure didn’t. She takes P’s assertion—that an effective official needs thesupport of a party—to mean that any official with party support will be effective. However,that which is necessary for effectiveness need not be sufficient for effectiveness, which isexactly what answer choice (E) tartly points out. All P is saying is that an independentcandidate—i.e. one without a party backing him or her—can’t be effective. But Q screwsthat up.

(A) Q does indeed offer evidence: it’s evidence that she believes to be a counterexample toP’s argument. The problem is that that evidence completely distorts what P has said aboutan independent candidate.

LSAT PREP ______________________________________________________________ LSAT Test XVII Explained: Section II

34 © K A P L A N

(B) focuses on a party’s possible decision to support an independent after an election, butthat has absolutely nothing to do with Q’s statement and hence cannot explain its weakness.

(C) As long as P and Q agree on a definition of “effective,” and there’s no reason to believethat they do not, neither the precision nor the vagueness of that term has any impact ontheir disagreement.

(D) is accusing Q of circular reasoning (that’s what “presupposing” one’s conclusionmeans). But far from presupposing P’s statement, Q is taking exception to it by presentinga counterexample.

• The confusion between necessity and sufficiency is at the heart of innumerableLSAT Logical Reasoning questions. Always be watchful for it.

12. (C)What do you know even before reading the stimulus? That it’s missing some crucialconcept or piece of information that’s needed for the conclusion to stand. How do youknow this? From previewing the question stem—always Step 1 of the Kaplan 4-StepApproach to Logical Reasoning. The phrase, “that is because,” in the second sentenceconfirms the first sentence as the conclusion and the second sentence as the reason, orevidence, for that conclusion. Try the “what and why” test: What does the author say?Public health will improve (an assertion) if medical researchers don’t wait to publish theirresults in peer-reviewed journals and instead inform the press as soon as the findingsemerge. Why will this happen? “That is because” people can use the information soonerrather than wait for the official publication. But that assumes (C); that people will usemedical information that hasn’t yet been published in the journals. If they won’t, theargument that public health will improve by abandoning the waiting period associatedwith publishing the findings completely falls apart. Thus, the Denial Test confirms that (C)is a logical, necessary assumption of this argument.

(A) and (B) Who cares who reviews the research and under what conditions they agree todo so? The argument centers on bypassing such peer review and getting the info out to thepublic.

(D) The author argues that medical researchers should abandon the process of waiting forpeer-review before announcing findings, so any info about the viability of fixing orspeeding up the peer-review process is simply irrelevant to the argument.

(E), if true, might strengthen the argument but is not an assumption that the author makes.(E) would support discontinuing publication in peer-reviewed journals but does not linkthe author’s evidence, the slow process, to that conclusion. Deny (E) and you’ll see that theconclusion could still be true—even if new medical information always receives publicattention (the denial of (E)), if it takes a long time to reach the public, then we should stillopt for the quicker straight to the press approach.

LSAT PREP ______________________________________________________________ LSAT Test XVII Explained: Section II

© K A P L A N 35

• The Denial Test works both ways: It can confirm a choice as correct if the negation ofthat choice leads to a crumbling of the argument, and it can also help you toeliminate wrong choices that, when negated or denied, do nothing or even supportthe argument.

13. (A)Now that we’ve understood the argument for question 12, it shouldn’t be too difficult tofind a weakener to rack up the point for question 13. The author argues that we shouldabandon peer-reviewed journals so that new medical discoveries can more quickly beused by the public to improve their health. In weakening this argument, we mustundermine the evidence for that conclusion. In this case we need to provide a reason whyexpediency isn’t in the public’s best interest. In (A), the journals serve some other, moreuseful function—they keep potentially dangerous and ill-formed medical conclusionsfrom reaching the street where they might negatively impact on public health. If (A) is true,a slow process is in the best interests of the public and we shouldn’t rush publication.(B) is consistent with the author’s argument; the fact that people act on the medicalinformation they receive through the press is almost a necessary component of theargument, but technically “alter their life-styles” may be a little extreme. In any case, in noway does this weaken the argument.

(C) This choice is outside the scope, plain and simple. This argument concernsimprovements in public health derived from new medical information, period; otherfactors are irrelevant.

(D), if anything, strengthens the argument by shoring up the viability of the author’s plan topublish quickly.

(E) The issue isn’t peer review or no peer review— the central issue of the argument iswhether the practice of informing the press of new medical findings only after peer reviewshould be abandoned. In other words, peer-review journals can still do their thing—this inand of itself doesn’t disrupt the argument, since peer review can still be performed after thepress has been notified.

• It’s not enough to understand everything the author says in his or her argument—when evaluating an argument’s logic, you must also think about what the author hasleft out or failed to consider. Pay careful attention to the discussion in the lessons,Previews, and Reviews regarding “alternative explanations /possibilities,” and be onthe lookout for the many other questions on released tests where this concept comesinto play.

LSAT PREP ______________________________________________________________ LSAT Test XVII Explained: Section II

36 © K A P L A N

14. (C)A paradox, of course, is a seeming contradiction, and at first blush it does seem as if thedecrease in wood workers contradicts the increase in wood yield over the same period.One possible resolution would be greater work efficiency on the part of the workers whoremained employed, but that doesn’t appear as an answer choice. The other resolution, theone we get in answer choice (C), is that the workers and the yield essentially have nothingto do with each other. Given that at least some of the 12,000 laid-off workers were woodprocessors, there’d be no paradox if the amount of unprocessed wood went up after thoseworkers left.

(A) Hearkening back to the 1950s does us no good, nor does the timber industry’s statusrelative to other industries. It’s all irrelevant.

(B) presents a different inverse relationship—acres down, demand up. In and of itself this isno paradox (demand ought to go up when acreage decreases), and since answer choice (B)doesn’t relate to the worker/wood yield issue, it doesn’t help us resolve the paradox we’vegot.

(D)’s domestic use vs. exported wood contrast is totally outside the scope, since the authornever alludes to different potential destinations for the wood.

(E) An increase in overall Ravonian unemployment that parallels the one in the timberindustry gets us no closer to resolving the dilemma presented in the paragraph.

• Trying to pre-phrase a correct answer to an LR question is generally a smart idea, andnever more so than in Paradox questions. A few seconds’ thought about how the twoseemingly contradictory phenomena can actually coexist will very often reap a quickand easy LSAT point.

15. (A)Anyone who read the question stem first would know to expect two things—the stimuluscontains a “principle,” and each choice contains a “judgment.” Our job is to find thejudgment amongst the choices that matches the principles in the stimulus. First, then, weneed to understand the principles in the stimulus. If a morally wrong act offends humanity(a principle), and all offenses against humanity are equally bad (a second principle), thenall morally wrong acts are equally bad. Now we can match the judgment in correct choice (A)to the principles in the passage: If lying is morally wrong and murder is morally wrong(the stimulus states that murder is morally wrong), these two acts must be equally badbecause the first sentence states that all morally wrong acts are equally bad.

(B) and (D) Risking one’s life to save lives is most likely a morally good act, while we don’tknow if accidentally killing someone is considered morally bad (for the sake of thisquestion, at least). These two choices therefore fall outside the scope of the argument,which concerns only morally wrong acts.

(C) Outside the scope—”prevention” goes one step too far and is an issue that’s neitherdealt with in the principles in the stimulus nor in the judgments of the choices.

LSAT PREP ______________________________________________________________ LSAT Test XVII Explained: Section II

© K A P L A N 37

(E) In order for (E) to conform, we’d have to infer, on our own, that not saving a life in sucha situation is a morally bad thing. This, however, would be unwarranted—the principles,taken together, simply state that any two things that are known to be morally bad can beconsidered equally bad. There’s no room in this principle for our interpretation of themoral value of various acts in specific situations.

• LSAT testmakers are big on definitions. Don’t enforce your own judgments on anevent if no definition is given, especially when all other events are strictly defined.For example, we don’t know where accidental death falls in the morally good / badspectrum; chances are somewhere in the middle. But notice how strictly all the actsof consequence (murder and lying) are defined. It’s not up to you to decide foryourself if accidental death (D), or for that matter failing to save a life by refusing tokill another (E), is morally good or bad; if the testmakers wanted you to know, theywould have told you. The ambiguity inherent in (D) and (E) should have scared youaway from these choices quickly.

16. (B)The stimulus conclusion is about “a majority” of members of a group; for that reason only(B) and (E), which deal with “most” members of a group, need be kept in contention.

You can perhaps understand the faulty logic in the stimulus best by picking a few easynumbers. Assume 100 voters. Suppose a majority—say 55 voters—vote for conservatives.Suppose that 55 voters vote for candidates supporting the antipollution bill. Do they have tobe the same 55 voters in both cases? Certainly not! In fact, it’s quite possible that no more than10 voters in one category fall into the other. (Example: 55 people vote for conservatives.The other 45 do not. Maybe all of those 45 vote for the candidates who support theantipollution bill—leaving 10 to make up the rest of the voters.)

Anyhow, the two majorities mentioned in the evidence need not make up the one bigmajority that the conclusion indicates....and by the same token, even though most kids maylike pie and most may like blueberries, the majority of kids don’t have to overlap as (B)’sconclusion suggests. There doesn’t have to be more than a small number of kids who likeboth pie and blueberries.

(A)’s logical flaw is its assumption that what can happen when gardens are tilled andplanted in wet soil (soil damage and plant rot, according to Bill and Sue) will happen. But(A) has nothing to do with necessary overlaps of majorities.

(C) commits two instances of mistaking necessity for sufficiency. No rain is a requirementfor Mark to go to the picnic, but it doesn’t assure his going; secondly, Mark’s attendance isa requirement for Susan’s going, but it doesn’t assure her going. (C) draws bothconclusions. But necessity vs. sufficiency is not the issue at the heart of the stimulus.

LSAT PREP ______________________________________________________________ LSAT Test XVII Explained: Section II

38 © K A P L A N

(D) only tempts because of its use of the word “majority.” The stimulus, remember, pointsto two separate majorities and concludes that they must overlap. (D), however, mentionsone majority—the majority that orders both fish and stuffed mushrooms—and concludesthat those are the dishes most frequently ordered. The flaw here is the assumption thatthose “who regularly eat” at the restaurant represent a huge number; after all, if the placehas very few regular eaters, maybe fish and stuffed mushrooms are ordered onlyoccasionally. But that’s not what’s going on in our stimulus.

(E) starts off promisingly, with two “majorities” at Gina’s house that need not overlap—those who cook well, and those who enjoy eating well-cooked meals. (E) fails by drawing aconclusion about a different issue: The nature of the meals served at Gina’s isn’t mentionedin the evidence. That’s a logical flaw, but it’s not the same as the stimulus.

• When two arguments are parallel, they draw the same kind of conclusion. Reject anychoice ((A), (C), and (D) here) whose conclusion deviates from the one in thestimulus.

• When a Parallel Logic question explicitly focuses on faulty logic, the wrong choiceswill either demonstrate proper logic, or logic flawed in a different way from that ofthe stimulus. (All four here happen to be of the latter type, as we’ve seen.)

17. (B)What’s wrong with the politician’s reply? The politician argues that the dispute over thedefinition of “wetlands” is “quibbling over semantics,” meaning that the definition doesn’tmake a real difference one way or another. But this is just his opinion; no evidence is givento support the notion that how the wetlands are defined is irrelevant—it may make a wholelot of difference to the critics of the bill. For example, the politician ignores the possibilitythat a restrictive definition might reduce the scope of the legislation’s impact. Thepolitician argues that the legislation would be more demanding in areas that aredesignated “wetlands,” but unreasonably dismisses the question of which areas willreceive that protection.

(A) distorts the argument. The politician does claim that the critics don’t really care aboutwetlands, but doesn’t make the further claim that the critics have the same motives as allthe opponents of conservation.

(C) Does the politician assume the critics will profit if the bill is defeated? No; that doesn’tshow up anywhere in the argument. In fact, the only thing the politician accuses the criticsof is not caring, and we can’t infer from that alone that he therefore assumes the critics willprofit from the bill’s defeat.

(D) No, it’s the critics of the proposed legislation who seem to prefer the less restrictivedefinition, and the politician has no duty to argue their case.

(E) is out of the scope. The author of the bill is never mentioned, and therefore his or hercredibility is not an issue.

LSAT PREP ______________________________________________________________ LSAT Test XVII Explained: Section II

© K A P L A N 39

• Focus on the scope of the argument, putting it in your own words. Wrong answerchoices often change the scope by distorting terms or adding new considerations.

• Don’t fault an author or speaker for omitting something from the argument that he’snot logically obligated to include. True, the politician doesn’t provide a defense forthe less restrictive definition, choice (D), but why would he?—in this case that wouldbe the burden of the critics.

18. (D)Keywords help you navigate your way through Dillworth’s unusually lengthy diatribe.Situation: People are deciding against kids for two reasons: A lot of sacrifice is required,and the kids end up ungrateful, grumble grumble. Contrast Keyword “However”: Hebrings up another consideration that, in his mind, outweighs the first: The fact that kids arethe best way of passing on one’s values. Conclusion Keyword “Therefore”: Dillworth’spoint is that those who hold values should separate themselves from the pack mentionedin sentence 1, and have kids despite all the sacrifice and ingratitude. (D) puts all of that inabstract language. The “category” is people with deep values, the “given course of action”is having kids, and the “reasons cited against [it]” are the sacrifices and ingratitude.

(A) is tempting to those who read choices too quickly, because it sounds superficiallyplausible. But Dillworth doesn’t deny that the sacrifices required to raise kids, and theresulting ingratitude, are real “drawbacks,” as (A) would have it. Instead, he takes them offthe table, saying that those drawbacks “have no bearing on” what should be the compellingreason for action, namely the desire to pass on one’s values.

(B) The choice Dillworth rejects—not having kids—he rejects only for those who hold deepvalues. And he doesn’t pass moral judgment on everyone else who opts against childrearing.

(C) Dillworth says nothing about prohibited actions.

(E) Just a lot of jargon—there’s no “specific-to-general” reasoning here. If anything,Dillworth argues that a course of action appropriate under one set of conditions would notbe appropriate under another.

• Proceed with care when answer choices traffic in abstract language. Such languagecan be interpreted just about anyway, unless you’re diligent in matching up thatlanguage to the specific case at hand.

• Beware of choices (like (B) here) that suggest that a speaker is impugning the motivesof others. Such answers are almost always wrong...and when they’re correct, youshould be able clearly to see the motive-doubting in the text.

LSAT PREP ______________________________________________________________ LSAT Test XVII Explained: Section II

40 © K A P L A N

19. (E)A majority of the time, when someone replies “You’re ignoring such-and-such,” we caninfer that the replyer is taking issue with the first speaker, and so it is with Travers here.She points out that whenever kids are ungrateful, it’s because they’re rejecting theirparents’ values—a fact totally and completely at odds with Dillworth’s claim that onereason for having kids in the first place is the opportunity to pass on one’s values. Answerchoice (E) puts it rather neatly: Dillworth has set aside the ingratitude issue to focus on thevalues issue. However, Travers points out that they are, potentially at the very least,interconnected.

(A) Travers doesn’t describe an alternative way in which kids acquire values, so (A) can’tbe her point.

(B) Self-sacrifice is the one aspect of Dillworth’s argument that Travers doesn’t concernherself with.

(C) Big distortion, (C)’s “well-known fact” notwithstanding, the rejection of values Traversdescribes is on the part of one’s children, not one’s peers.

(D) goes too far. Travers isn’t saying, Hey, all kids reject their parents’ values, so don’t evenhope to pass yours along through your progeny. She simply argues that Dillworthshouldn’t be so quick to ignore some of the reasons people use to opt out of child rearing.

• When two people are arguing—on the LSAT, at least—be sure to paraphrase whateach is saying, and to explore the relationship of the second person’s comment tothat of the first. Questions usually hinge on those two issues.

20. (B)We’re looking for an inference, a statement that must be true based on the statements in thepassage. In such questions, the answer can come from anywhere in the passage, and wemay not even know beforehand what part of the passage to specifically focus on. Evenisolating the conclusion may not help greatly, since the right answer to a strict inferencequestion like this could come from a seemingly obscure piece of evidence, or even anirrelevant aside. In this case, correct choice (B) is inferable solely from the second sentence:If the feeding methods of jawless fishes were “limited to either sucking in surface planktonor sucking in food particles from bottom mud,” then we can be 100% sure that thesejawless fishes didn’t prey upon other fish. As simple as that—which is not to say it’sobvious. Many test-takers passed on (B) because it seemed outside the scope, or just notimportant enough. But sure enough, the passage could end with the word “mud” insentence two and (B) would still be inferable—it may get lost in the verbosity andonslaught of facts in the rest of the passage, but there’s no denying that fish who’s foodchoices are LIMITED to sucking EITHER plankton OR mud particles surely aren’t preyingon other fish. (For more on the nature of this strict inference question type, see the firstbullet point below.)

(A) The only thing we’re told about jawed-fish in relation to prey is that they’re able topursue prey, and use their jaws and teeth on it. What kind of prey we’re talking about hereisn’t mentioned, so there’s no way we can infer it’s primarily fish.

LSAT PREP ______________________________________________________________ LSAT Test XVII Explained: Section II

© K A P L A N 41

(C) Why not? How do we know this? We absolutely don’t. Nowhere does it say that jawlessfishes had a monopoly throughout history on bottom mud particles—teleosts can partaketoo.

(D) This is simply answer choice (C) in reverse—just because some jawed fishes had orhave cartilage as their skeletal material doesn’t mean that jawless fishes did not; maybe theydid.

(E) No; 400 million years ago is the date given for the first development of biting jaws infishes, but we have no reason to believe that the entire population of jawless fishesdisappeared instantly on or around that date. That is, nothing in the stimulus precludesthe possibility that some classes of jawless fishes remained alive for quite some time whilejawed fished evolved from other classes of fish.

• Note the wording of this classic type of inference question—it doesn’t ask for thechoice that’s most strongly supported, as do some inference questions; no, it asks forthe choice that must be true based on the stimulus. The latter is in fact much stricterthan the former; it implies that somewhere in this long narrative exists a premise or acombination of premises that leads to an airtight deduction, just like in LogicGames. The flip side of this is that the answer may not sound very important at all,but that’s okay; we’re not looking for the main point, or a central assumption that theargument depends on. The correct answer to such a question can be and often is assimple and innocuous as (B) here, derived from the seemingly unimportantinformation in the second sentence. In this case, every sentence besides the secondsentence exist merely as fodder for wrong choices. Any time a question stem takesthis form, be aware that the answer can come from anywhere, and from the simplestinformation no matter how complicated the passage looks or how long it is.

• Sometimes, the most effective method is to simply check the choices against thepassage. It would take you a long time to pre-phrase the correct answer to aninference question like this. If you get the feeling that a question may not be ripe forpre-phrasing, skim the passage once quickly to get the gist, and then spend the bulkof your time carefully evaluating the choices.

• Revisit the concept of “what DON’T you know?” from the lessons, previews andreviews. Those were exercises that pointed out how important it is to recognize whatwe CANNOT infer from stimuli in order to eliminate wrong choices.

LSAT PREP ______________________________________________________________ LSAT Test XVII Explained: Section II

42 © K A P L A N

21. (C)We know from the stem that we’re looking for an assumption, so it’s wise to be on thelookout for the gap in the logic in our initial reading of the stimulus. The author concludesthat in “geologically quiet” regions, the best way to reduce the risk of having a nuclearreactor struck by an earthquake is to place it near a fault that has produced an earthquakewithin living memory. As evidence, the author notes that such faults cannot produceearthquakes more often than once every 100,000 years. So, if such an area has experiencedan earthquake within living memory, which presumably means no more than 80 or 90 yearsago, the next one must be at least around 100,000 years away. This is a low risk, but it’s stillhigher than no risk at all. If there were potential sites that were distant from even minorfaults, then they would be better choices. In other words, the author assumes that everyreactor site in such a region must be next to a fault, which may not be the case. If it’s not,then the most earthquake-proof place for a reactor site in a geologically quiet regionneedn’t be where the author suggests. We can thus deny choice (C) and see the argumentfall apart, which confirms (C) as the assumption we seek.

(A) is too broad. The argument concerns the danger posed by earthquakes, and not dangerin general, so the argument doesn’t rely on a danger comparison of different types ofregions.

(B) The argument needs no help from information about whether earthquakes constitutethe main safety concern for reactors; for all we know, there are other more importantdanger factors involved. Either way, this issue is irrelevant to the argument, which is solelyconcerned with the prospects of an earthquake striking a reactor in a specific region.

(D) No; the issue is specifying the site with the least chance of getting struck by anearthquake—what happens if one does get hit is irrelevant. Therefore, the notion of howmany earthquakes such a reactor can withstand has no bearing on the argument at all.

(E) The evidence states that such faults produce earthquakes no more than once in 100,000years. If we change that to at least once in 100,000 years (as in choice (E)), then placing areactor near a minor fault becomes slightly more risky, which doesn’t help the author.

• Noticing scope shifts between the argument and the choices will help you eliminatethe wrong ones. To do this, keep the crux of the conclusion in mind. Here, the authoris primarily arguing about the location in a particular type of region that is “leastlikely to be struck by an earthquake.” If you focus on that sentiment throughout,you should be able to recognize the subtle shifts away from it contained in choicessuch as (A)—where overall are the least or most dangerous places to build reactors;(B)—what part do earthquakes play in the overall safety of reactors; and (D)—howare reactors able to withstand earthquakes? Since all of these issues are beyond thescope, they have no bearing on the argument and therefore cannot be assumed bythe author.

• Once again, we see an assumption based on an alternative possibility. Inassumption, strengthen/weaken, and logical flaw questions especially, be on thelookout for other possibilities / conclusions / explanations that the author overlooks.

LSAT PREP ______________________________________________________________ LSAT Test XVII Explained: Section II

© K A P L A N 43

22. (D)The first task in this one is to make our way through the complicated stem. Boiled down toits basics, the question simply asks us to assess the second argument as a response to thefirst. In other words, does it work or not, and why? Notice that the first three choices beginwith “it succeeds” and the last two start with “it fails.” That makes it easier; we can narrowthe choices down first by a gut reaction to the validity of the response, and then go on topinpoint the choice with the correct reason.

Well, did you think the response worked? Hopefully not; the Advertising-Sales directormisses the point. The Magazine editor is worried that readers will perceive a decline ineditorial integrity on the part of the magazine if the magazine favorably mentions productsin articles. The Advertising-sales director counters “don’t worry about it . . . people knowthe difference between articles and ads, so the response to the ads has nothing to do with themagazine’s editorial integrity. . .” Here’s where you should have begun scratching yourhead and muttering “huh??!! . . . what does that have to do with anything?” That one quickgut feeling is enough to kill choices (A), (B), and (C)—no way does this response succeed,it’s barely talking about the same thing. The editor is talking about a reader response toarticles and the director is talking about a reader response to ads.Now to narrow down the precise reason why the response fails. The editor doesn’t care, atleast in this argument, about readers’ response to ads—she’s worried about their responseto the articles written to glorify the advertisers’ products. (D) cites this as the correct reasonwhy the response fails as a counter to the editor’s concern. As for (E), the editor doesn’tstate a view about reader response to advertisements, so obviously the advertising-salesdirector can’t misunderstand it.

• When you have a good grasp of what’s going on in the passage, and an idea of whatthe right answer will look like, scan the choices instead of reading them all in depth.You should have eliminated (A), (B), and (C) quickly after reading the first twowords of each. If you got the immediate sense that something was wrong with theresponse, the only choices worthy of evaluation were (D) and (E).

• Beware of the subtle scope shifts those wily testmakers throw into the variousarguments. These types of errors in logic are not entirely outside the scope; instead,they usually involve an author or character having an entirely wrong perspective onthe topic at hand.

23. (C)In all assumption questions, we are looking for an answer choice that helps to link theevidence to the conclusion. For Question 23 we can ignore the Advertising-sales director’sresponse because we are only asked about the editor’s argument. The editor’s conclusion isthat the magazine shouldn’t endorse advertisers’ products in its articles because doing sowould actually hurt the advertisers, not help them. The evidence for this conclusion is theeditor’s predicted loss of readership. The assumption is that the loss in readership willsomehow hurt the advertisers (relatively speaking) despite the product endorsements theyare getting in the articles. In other words, the editor assumes that the benefits from productendorsements in articles (and corresponding loss of readership) are worth less toadvertisers than the benefits they get from their current advertisements, or as answer choice(C) states: favorable mention in the articles is of less value than the current advertising.

LSAT PREP ______________________________________________________________ LSAT Test XVII Explained: Section II

44 © K A P L A N

(A) In this one particular case, the editor feels she must go against the wishes of some ofthe regular advertisers, but there’s no reason to believe that, in making this argument, theeditor relies on a policy of never considering advertisers wishes. In fact, the editor appearsto fight for the best interests of the advertisers, behavior that strongly implies that she doestake their wishes into account.

(B) is actually part of the editor’s evidence for her conclusion; it’s not an underlyingassumption on which the argument relies.

(D) Au-contraire—if the editor believed this, her argument would be totally different; shewouldn’t believe that granting the advertisers’ wishes would work against them.

(E) This sentiment does nothing for the validity of the argument. Just because the editorfeels that product endorsements in articles may cause problems of integrity and loss ofreadership, there’s no grounds to believe that she assumes that normal paid advertisementsare never problematic themselves. However she feels about this issue, it’s not crucial oreven relevant to this particular argument.

• In a classic assumption question, such as 23, always identify the evidence and theconclusion and pick the answer choice that best links the two together.

24. (A)The Keywords “For example” immediately signal that the author’s conclusion appearsearlier, and so it does: It’s that arguments employing statistics don’t advance policydebates, but instead “deflect public fears.” That’s a little obscure; did you take that conceptand push it up against the given example? If so, you probably noticed that, to the author,the policy debate over auto safety isn’t especially advanced by the seat belt statistics;instead, those stats “deflect public fears” by making them feel safer when they buckle up.(And inferably, may encourage people to drive more, thus increasing the safety risks.) In thesame way, (A)’s stats about the relative safety of nuclear plants don’t advance the debateover technology and health; they may even blind the public to the real issue ofconservation vs. nuclear power, and encourage the latter.

(B) “These statistics counter [a] widely-held...belief”: The seat belt stats aren’t used to“counter” anything; they’re used to show how statistics can undermine public policydebates.

(C) is an argument for not using statistical arguments as the sole basis for the public policydebate. Nothing like this is in the stimulus.

(D) cites life expectancy statistics, and then cites other facts that cast doubt on the statistics.No relationship to the stimulus at all.

(E), to its credit, does indeed mention public policy, but answer choice (E) shows howstatistics (about people’s various addictions) can actually advance the public debate (aboutprograms to fight substance abuse). This is more or less the exact opposite of the author’spoint.

LSAT PREP ______________________________________________________________ LSAT Test XVII Explained: Section II

© K A P L A N 45

• Be careful of bringing your everyday-life biases to bear on your understanding ofLR arguments. Here, for instance, we mustn’t let our conviction that seat belts are agood thing blind us to the author’s view (that focusing on seat belts distracts fromthe real policy debate over auto safety).

25. (D)Since W draws no conclusion, the facts she cites make sense only in the context of S’sargument that we shouldn’t try to halt future global warming because scientists disagreeall over the place; the evidence isn’t firm; global warming may not even be real. What Wdoes is suggest that scientists don’t bother to argue when facts are established, and thatscientists do accept one fact: Global warming is happening, albeit within an uncertain 3°Crange. W’s facts are the “similarities” among scientists to which (D) refers.

(A) W and S are equally aware of the disagreements among scientists; it’s just that Wdenies those disagreements’ relevance to the global warming debate.

(B) Since W doesn’t bring out her explicit point of view, we can’t be sure that W accepts S’sconclusion—indeed, their profound disagreement over the global warming trend impliesthat their conclusions would be very different.

(C) Inferably, W’s scientists are the same as S’s. They just look at these authorities fromdifferent perspectives.

(E) Another accusation of circular reasoning, but this one is balderdash. (See also Q. 2 (E)and 11 (D) in this LR section, and Q. 20 in the other.) First of all, W doesn’t explicitly offer aconclusion at all; also, her evidence is quite concrete with no hint of “circularity.”

• It’s always a good idea to read the second speaker’s comment first (doing so is clever,and saves time). Just be prepared to need, and rely on, the first person’s remarks ascontext for understanding the overall disagreement.

46 © K A P L A N

SECTION III:

LOGICAL REASONING

LSAT PREP _____________________________________________________________ LSAT Test XVII Explained: Section III

© K A P L A N 47

1. (B)The stem tells us to draw a conclusion from the formal logic-type statements found in thestimulus. The major key is for you to recognize that the first statement is a statement ofnecessity: “there can be no growth without investment” means that if capacity is full,growth needs, or requires, investment. That is, under full capacity, investment is necessaryto bring about growth, although there may be other factors that are needed as well. Nowwe can integrate the second sentence: Reducing interest rates produces investment, thevery thing we just saw is necessary for growth. Bottom line: Reducing rates won’tguarantee growth, but it will lead to one necessary factor for growth—investment. In otherwords, as answer choice (B) has it, a reduction in rates allows for a condition necessary forgrowth to come about.

(A) contradicts the stimulus. Any reduction in interest rates produces new capitalinvestment—no exceptions.

(C) is an out-of-place policy recommendation. The argument never discusses what shouldbe done with interest rates or anything else.

(D) There are no restrictions on the source of new capital investment. The stimulus dealsonly with falling rates; since we’re told nothing of rising rates, it’s totally possible that newcapital investment that takes place while interest rates are rising could lead to industrialgrowth.

(E) This answer choice scrambles the terms of the argument. The argument never mentions“manufacturing capacity newly created” and there is no requirement that it be “fullyutilized.”

• Reading the question stem first tells you the task that you are assigned and directsyour attack on the stimulus. In this question, you’re told to draw a conclusion bycombining statements.

• In formal logic questions, don’t be intimidated by jargon like “fully utilizedmanufacturing capacity.” You don’t need to understand the theory of economics tofind the answer. You do, however, have to paraphrase the argument, and shorteningthe major terms to capacity, growth, and investment could make the task moremanageable.

• Beware of the “can vs. should” scope shift, seen here in answer choice (C). If theargument is only in terms of what is possible, then no conclusion can be drawnabout what ought or should be done.

LSAT PREP _____________________________________________________________ LSAT Test XVII Explained: Section III

48 © K A P L A N

2. (D)As soon as you see “apparent discrepancy” in the stem, you know what to expect:Something in the passage is going to seem odd—an unusual circumstance, a strangeresult—and it’s up to you recognize the choice that clears the whole thing up. Theapparent discrepancy isn’t too hard to spot: Several traps, and the number of rose beetlesdecreases—so far so good. Only one trap, however, and the number of little buggersincreases. Huh? It’s odd, it’s strange, we’ve located the apparent paradox. You’d think thatone trap would reduce the number of beetles, just not by as much as several traps. Youmay not be able to precisely pre-phrase an answer, but you should be able to get a feel forwhat the right answer will sound like: There’s something about the mechanism of this trapthat makes them work well in bunches but horribly by themselves. That’s the only notionyou need to recognize answer choice (D)—throw any trap down, even just one, and morebeetles than one trap can handle are lured to the site. That explains why using only onetrap will cause a net increase in beetles. But using several traps won’t attract many morebeetles than one trap will, and now the capacity for catching the little guys is increased.Answer choice (D) explains why using several traps together does the job, while using justone trap alone makes the problem worse.

(A) not only fails to explain the given paradox, it actually adds to the mystery: Why wouldmore beetles result from using one trap if the beetles can’t even detect the scent of a singletrap?

(B) makes logical sense; one would expect several traps to have this kind of advantage overa single trap. But if you chose answer choice (B) on this basis alone, you forgot whatquestion you were answering. (B) shores up one element of the stimulus (use several traps,and the number of beetles will be greatly reduced), but entirely ignores the crux of thediscrepancy—we’re still in the dark as to why laying down a single trap increases thenumber of beetles.

(C) Here’s information that like (B), stands to reason, yet does nothing to erase the mysteryin question.

(E) Okay, in this scenario, beetles check in, but some DO check out (remember that roachmotel commercial?). How does this lead to an overall increase in the number of beetles inthe rose garden? This choice shows promise, but doesn’t go far enough to make a necessaryconnection that allows us to say “aha! that explains it!”.

• Be wary of choices in paradox questions that sound reasonable (like (B) in this case)simply because they’re in accordance with some of the information in the stimulus.That’s not enough; a good choice must still make the seemingly contradictoryelements of the stimulus mesh.

LSAT PREP _____________________________________________________________ LSAT Test XVII Explained: Section III

© K A P L A N 49

3. (D)This is a standard assumption question; our task is to locate the evidence and conclusion,and then to find the necessary but unstated premise that connects them. The conclusion issupplied by the first sentence, which states that computer programs should not bepatented. For evidence, we’re told patents should not grant large corporations control overa methodology, and computer programs are implementations of methodologies. If theauthor believed that large corporations should hold patents for implementations ofmethodologies—the negation of choice (D)—then the argument would crumble.Obviously the author feels otherwise, which means that (D) is necessary to the argument,and must be assumed.

(A) weakens the argument. If patents are designed to protect small-time inventors fromexploitation, then the development of computer programs by small-time inventors wouldtend to support the move to patent computer programs, which is the opposite of theauthor’s view.

(B) is irrelevant, which can be demonstrated by trying the Denial Test: If implementing amethodology requires more creative effort than does true invention, would the argumentbe damaged? No, so answer choice (B) isn’t necessary, and cannot be a necessaryassumption.

(C) is also irrelevant—whether the issue of patents for computer programs has been raisedin the past is a totally different issue from the argument at hand, which focuses on thedesirability, not the history, of program patenting.

(E) Again with the small-time inventors . . . Small-time inventors are mentioned only inrelation to the origins of the patent system, while most of the argument focuses on therelation between computer programs, patents, and large corporations. What small-timeinventors believe as a group, and how their beliefs impact on their cause, are issues foranother argument altogether, not this argument. This argument needs none of this tosurvive.

• The Kaplan “Denial Test” builds strong LSAT scores in at least two ways. First, it isextremely useful as a backup, to confirm that your choice in an assumption questionis in fact necessary to the argument. If the argument could stand without a particularchoice, then that answer choice isn’t necessary. Second, it’s sometimes hard to seewhy a choice is an assumption, yet easier to see why the negation of the choicewould defeat the argument. In this case, it might be difficult to see why the argumentdepends on answer choice (D) until you think about what would happen if answerchoice (D) were false.

LSAT PREP _____________________________________________________________ LSAT Test XVII Explained: Section III

50 © K A P L A N

4. (D)Question 4 asks, “How does Larissa make her argument?,” Question 5, “What is a point atissue between Larissa and Walter?” Start with whichever question you feel morecomfortable. Walter’s conclusion is that tolerating an injustice is morally wrong andshortsighted. His evidence follows the colon (always pay attention to punctuation). Larissaalso believes that allowing injustices is bad policy, but disagrees with Walter as to thereason. The key to understanding Larissa’s evidence is the phrase “not because . . . butbecause.” She disagrees with Walter’s reason for the conclusion and provides her own.Therefore, you’re looking for an answer choice, such as (D), which states that Larissa agreeswith Walter’s conclusion but disagrees with his evidence (reason).

(A) might have been tricky if you didn’t read the stimulus critically. Larissa agrees withWalter’s conclusion, she only disagrees with his reasoning.

(B) Larissa’s conclusion is the same as Walter’s, and she bases it on her own evidence—shedoesn’t do anything with any assumptions Walter may have, let alone draw implausibleconsequences from them. (She does disagree with his evidence regarding the wealthy’svulnerability to injustice, but that’s another matter.)

(C) Larissa deals only with Walter’s argument, she never questions Walter himself or hisauthority to address matters of social policy. In order for (C) to be correct, Larissa wouldneed to have said something like “Walter is not in a position to make his assertions becausehe is neither an economist nor a politician.”

(E) Although Larissa believes that the possibility of social unrest is the reason that weshould not tolerate injustice, she does not mention a belief that Walter should have morefully developed his argument. She simply disagrees with his rationale.

• In method of argument questions, it’s often helpful to try to imagine what thestimulus would have to sound like if the choice you’re analyzing were correct. Asnoted in (C) above, the stimulus would have to be much different than it is for thischoice to describe Larissa’s method, which allows us to confidently cross it off.

5. (E)As the analysis for Question 4 shows, Larissa and Walter agree on the conclusion, butdisagree on the evidence supporting that conclusion. Walter believes that we should nottolerate injustice because it may affect a well-to-do person tomorrow. Larissa believes thatwe should not tolerate injustice because it’s a source of social unrest. Larissa doesn’tmerely present different evidence however, she affirmatively disagrees with Walter’sreasoning. Larissa states, in the first sentence of her response, that the well-to-do can protectthemselves against injustice better than the rest. Walter believes that injustices can inflictthe well-to-do just as easily. So, Walter and Larissa disagree on whether the economicallyprivileged members of a society are less exposed to certain sorts of injustices than are theeconomically disadvantaged—answer choice (E).

LSAT PREP _____________________________________________________________ LSAT Test XVII Explained: Section III

© K A P L A N 51

(A) Social fabric? Hopefully, this new term immediately caught your attention. AlthoughWalter and Larissa do indeed disagree on how injustice affects the privileged and thedisadvantaged, they never debate whether the rich and poor are part of the same socialfabric.

(B) This answer choice distorts several things that are mentioned. Inferably, both Walterand Larissa believe that at least some economically privileged people “tolerate,” that is,allow, injustice against the disadvantaged. Whether this makes these rich folks the “leasttolerant people” in the society is ambiguous at best. And even if you did make this leap,which is unwarranted, you’d have to infer this judgment for both Walter and Larissa; thatis, it wouldn’t be a source of conflict between them, so it still can’t be the point at issue weseek.

(C) is an au contraire answer choice. Both Walter and Larissa agree that the disadvantagedmembers of society suffer from injustice, and they both agree that this is bad. They disagreeon why it is bad.

(D) Neither Walter nor Larissa discusses who is responsible for correcting social injustices.They both agree that social injustices are bad, they differ on why injustices are bad, butneither mentions what should be done to fix the problem.

• Use common sense: We’re told in the stem that Walter and Larissa are logicallycommitted to disagreeing about something, but since Walter and Larissa agree on theoverall point (allowing injustice to persist is bad), you should have realized that theanswer to this question is buried in the evidence.

• If you find a question time consuming, make a good guess after eliminating one ormore answer choices, and move on. There may be easier questions to come. If timepermits, return later.

6. (E)The market analysts predict that the decision of leading manufacturers to concentrate theirdetergents will lead to the virtual disappearance of the older, bulky style. Theconcentrated detergents will be sold in smaller packages that will save production costs,which gives manufacturers a reason to prefer the new approach. This is all well and good,but the author does neglect a full half of the supply and demand equation—what about theconsumers? If the consumers ain’t going to buy the new detergents, the manufacturers, andthis argument, are sunk. If, on the other hand, the consumers had a reason to prefer the newapproach, then the argument would be strengthened. Answer choice (E), proposing thatconsumers would prefer concentrated detergents due to environmental concerns, fits thebill.

(A), if anything, weakens the argument. If smaller manufacturers were unable to adopt thenew approach, then the eventual disappearance of bulky detergents would be much lesslikely.

LSAT PREP _____________________________________________________________ LSAT Test XVII Explained: Section III

52 © K A P L A N

(B) and (C) Both of these also lean more towards weakening than strengthening theargument; both paint a negative picture of the new detergent/consumer relationship.Initial skepticism about the effectiveness of the concentrated detergent answer choice (B)could only weaken the argument. However, since the argument refers to the eventualdisappearance of bulky detergents, initial skepticism wouldn’t damage the argument toomuch, making this choice mostly irrelevant without additional information. As for answerchoice (C), if the analysts are correct and consumers have to pay more to use theconcentrated style, the law of demand suggests that bulky detergents would be more likelyto stay on the market.

(D) A discount from major supermarkets would have been a boost to the argument.However, if supermarkets charge the same for shelf space for the new product, then it’shard to see how this would affect the argument at all.

• Don’t rush through the question stem. If you do, you might mistakenly identify astrengthen question as a weaken question, or make a mistake about the particularargument that needs to be evaluated. If you do get it backwards, then an au contrairechoice (in this case, one of the many that tends to weaken the argument) might looktempting.

7. (C)Since the question stem tells us that the critic believes that the advocate’s argument isflawed, match the critic’s language and logic to the relevant portions of the advocate’sargument. The critic suggests that contribution caps (the advocate’s second reason) willprevent the candidate from spending more time serving the public (the advocate’s firstreason), because candidates will need to spend more time finding more small contributors.As (C) states, the two projected results (more time with the public and less time workingfor large contributors) cited in support of the proposal (public campaign funding) workagainst each other.

(A) is outside the scope; the critic’s objection never mentions large contributors or ways tocircumvent caps. Don’t read more into an answer choice than you’re allowed. That canlead to missed points.

(B) is a tricky half-right half-wrong answer choice. It starts out strong by mentioning bothresults in support of the proposal, but fades quickly when it states that the two reasons areactually one. The critic believes that the two reasons are inconsistent, not that one entailsthe other.

(D) mentions something, the possibility that large competitors will stop contributing, thatneither the advocate nor the critic mention. Thus, (D) can’t be a flaw in the critic’sargument.

(E) sets up a false contrast/distinction that isn’t present in the critic’s response. The critic’sresponse does not mention generous contributors or moderately generous contributorsand never compares the two.

LSAT PREP _____________________________________________________________ LSAT Test XVII Explained: Section III

© K A P L A N 53

• When the response in a dialogue stimulus is substantially shorter than the initialstatement, consider starting with the shorter, and often more manageable, secondstatement first.

• Always read the entire answer choice. Half-right half-wrong answer choices oftenfool the uncritical reader.

8. (A)According to the advocate, there are two reasons campaigns should be subsidized withpublic funds: 1) incumbents will be able to spend more time serving the public, and 2) itwill be less likely officials will work for large contributors. Both reasons cited are onlybenefits to the public if public funds are used. Neither are as likely, according to theadvocate, if private funds are used. Answer choice (A), in a wordy sort of way, says justthis.

(B) Hopefully, when you scanned all five answer choices quickly, you dismissed answerchoice (B) immediately as being outside the scope of the argument. Neither the advocatenor the critic ever discusses lengthening the terms of office for elected officials in theirarguments.

(C) Neither the advocate nor the critic discusses subsidizing under-funded candidates.Both talk in generalities about rules that should apply to all candidates. (C) sets up acontrast that doesn’t exist in the stimulus.

(D) is a long and convoluted answer choice, so rather than trying to deal with the entirechoice all at once, try breaking it down clause by clause. The first clause talks aboutbenefits for specific individuals—the stimulus never mentions specific individuals andthis alone would allow you to eliminate the choice. If you couldn’t decipher the firstclause, don’t be afraid to start with the second clause. The second clause (after the comma)says individuals should fund the activity (political campaigns). This runs counter to theadvocate’s argument that the public should fund campaigns, and gives us another reasonto axe choice (D).

(E) The advocate never discusses who would run with and without public campaignsubsidies, merely that all campaigns should be subsidized with public funds. Thus (E) isoutside the scope of the argument.

• In a principle question, the credited response is a restatement of the author’sargument, usually in more general terms. Answer choices that depart from the termsof the argument are normally wrong.

• In principle questions, translate the abstract elements of the answer choices to thespecific elements of the stimulus.

LSAT PREP _____________________________________________________________ LSAT Test XVII Explained: Section III

54 © K A P L A N

• Process of elimination might have been your best approach to this question. Youprobably quickly eliminated some combination of answer choices (B), (C), and (E).Answer choices (A) and (D) might have caused you a little more trouble because oftheir length and abstract wording. If a question seems tricky, and you can quicklyand confidently eliminate a few choices, guess and move on. If time permits, you canreturn to it later.

9. (D)The bird-watcher proves that he knows no more about logic than he does about animaltracks when he identifies a typical trait of birds (three toes forward, one back), and thenconcludes that a track with that trait must be that of a bird. That’s like saying that since mostsailors wear a white hat, any particular person with a white hat must be a sailor. As if! Now,if we knew that sailors and only sailors wore a white hat, then that conclusion would becorrect; likewise, the stimulus conclusion would work if we knew that only birds madetracks in that 3/1 pattern. But that evidence is absent. Answer choice (D) points out theflaw.

(A) As used, “track” is adequately concrete.

(B) The bird-watcher takes for granted that birds are animals, and that “typically” theyhave four toes. No further definition is required.

(C) Since the issue is simply whether the animal that made the track in question was a birdor not, the type of bird is irrelevant.

(E) has it exactly backwards. The evidence is about birds in general, and the conclusiondrawn is about an individual critter.

• Try to pre-phrase an answer to Logical Flaw questions whenever possible. Doing soshould help you avoid wasting time on implausible answer choices. Wasted time canlead to missed points later in the section.

10. (D)The stimulus begins with the psychologists’ claim that people are more susceptible topsychological problems in the winter than in the summer (seasonal affective disorder). Theonly support cited for this claim comes from the survey. The psychologists are assumingthat the self-reported results of the survey are accurate. The author disagrees with thisassumption and uses the word “however” to signal that she disputes the validity of thesurvey results because she does not believe that self-reported results about the past areaccurate. The author, “therefore,” does not believe that the survey results support theexistence of the disorder. The author disputes the psychologists’ conclusion byquestioning their central assumption that the results of such a survey can be accurate—choice (D).

LSAT PREP _____________________________________________________________ LSAT Test XVII Explained: Section III

© K A P L A N 55

(A) The author believes the survey results are suspect, but she never ventures her ownexplanation of the change in susceptibility to psychological problems across seasons. Infact, in the end she denies that the disorder exists, so why would she offer an explanationfor it?

(B) is way too broad: Our author questions the existence of such a disorder because sherejects the methodology of this particular survey. That’s not the same as her questioningwhether any such variation, under any circumstances, could be labeled a disorder—thatwould be a few steps beyond this specific argument. In other words, the issue to the authorisn’t whether or not such variation across seasons in psychological states constitutes adisorder, as (B) would have it; she out-and-out denies the existence of the variationaltogether because she doesn’t trust the survey results.

(C) Although the author does not believe that the survey supports the psychologists’conclusion that the disorder exists, she never questions whether or not the survey sample isrepresentative—she instead questions the survey respondents’ ability to accurately recallhow they felt.

(E), like (A), implies that the author believes in the existence of seasonal affective disorders,but we know that the author really feels that the survey evidence doesn’t justify the claimthat such a condition exists.

• Learn to use structural clues such as “however” and “therefore” to help you tounderstand the way in which an author makes her argument. Much moreimportantly though, they help you to answer questions quickly and accuratelyduring your test day.

• Scope shifts take many forms on the LSAT, some of them can seem particularlyamusing. Answer choice (A) has the author arguing over the reasons for theexistence of a phenomenon she doesn’t even believe exists, and answer choice (E)has her quibbling with the psychologists over the number of existing cases ofsomething she doesn’t believe exists. Both choices are silly; their logic is comparableto a person complaining “I don’t like this restaurant because it doesn’t serve frenchfries AND because their french fries are too greasy.” Spotting such obviousinconsistencies between the stimulus and the choices will help you to eliminatewrong choices with greater ease.

11. (B)The argument is a chain of cause-and-effect—a prediction of what the author believes willhappen if the city gets rezoned: New water and sewer systems, and then new apartmenthouses will be built; schools will become overcrowded; overcongested roads will lead tonew roads. We learn in the next to last sentence that a bigger tax bite will be needed forthose civic improvements. Hence answer choice (B) is a proper inference: If the newapartment buildings (a result of the rezoning) are built, then the tax bite (a necessarycondition for the civic improvements that will follow those apartment buildings) willincrease.

LSAT PREP _____________________________________________________________ LSAT Test XVII Explained: Section III

56 © K A P L A N

(A) is a classic distortion. “Unless [they] band together, the [rezoning] will be approved”means that in the absence of the citizens banding together, the rezoning will go through. Wecannot, however, presume that that banding together would necessarily halt the rezoning(and all the subsequent developments). Maybe they would all go through anyway.

(C), (D), and (E) The stimulus more or less argues that if the rezoning plan goes through,Glen Hills’ rural atmosphere might be destroyed (by the sequence of events described). Weare not, however, permitted to assume that the failure of the plan will ensure thecontinuation of that rural atmosphere—but that’s just what (C) does. Likewise, if theapartment buildings are built, taxes will go up; but (D) just denies both terms. That’slogically forbidden, as is (E)’s inference that not building the apartment buildings willresult in no overcrowded schools or congested roads.

• “If X, then Y” cannot be rewritten as “If not X, then not Y.” The fallacy of denying theantecedent is no academic exercise; it’s something that happens in life, and on theLSAT, all the time. Watch out for it.

12. (D)We need to strengthen the argument that the advertising campaign was responsible for thereduction of the number of smokers—a pretty hasty conclusion, considering there’s anobvious competing explanation: cigarette prices are now 20 cents higher. It’s not going outon a limb to speculate that this factor might have played a role in the decline. In makingher conclusion, the author assumes that the 20 cent increase played no part in the smalldecline in smoking, which would be the answer if this were an assumption question. But asyou know from class and the Kaplan Previews and Reviews, a central assumption couldform the basis of a strengthen or weaken question as well. Break down the assumption, andyou have a weakener; shore up the assumption, and you have a strengthener. (D) does thelatter: If merchants responded to the tax by lowering the price of cigarettes by the exactamount of the tax, then a price increase drops out as a candidate for the decline, theauthor’s central assumption is validated, and the argument that the advertising was thecause of the decline is strengthened.

(A) is outside the scope. The argument deals with smoking, not tobacco consumption ingeneral.

(B) is also outside the scope. The author claims that the number of smokers has beenreduced, and so smokers that are merely cutting down don’t count.

(C) provides very weak evidence for the notion that the number of smokers has declined,but does not address the cause of the reduction.

(E) is mostly neutral, but one could argue that if smokers are relatively poor, then theymight be sensitive to the cigarette price increase. The increase would then have more of animpact than the author acknowledged, which would tend to weaken the argument. In anycase, whether this choice slightly weakens or is neutral to the argument, you shouldn’thave picked it as a strengthener.

LSAT PREP _____________________________________________________________ LSAT Test XVII Explained: Section III

© K A P L A N 57

• The ability to recognize assumptions is crucial, not only for assumption questions,but for other question types as well. This is what we mean when we at Kaplan talkabout the synergy that exists between Logical Reasoning question types. Whenlooking for ways to strengthen and weaken arguments, picking up on an author’sunstated central assumption, if one exists, is more than half the battle.

• When there are two competing explanations for an event, and you’re asked tosupport one over the other, a choice that mentions neither, or applies equally well toboth, must be wrong.

• Scope shifts everywhere!! Choice (B) tries to persuade you that the amount thatpeople smoke is somehow important, when in fact the only relevant issue here is thenumber of smokers.

• Beware of answer choices that attempt to strengthen a fact in the stimulus rather thanthe argument the author is trying to make. For example, answer choice (C) could betaken as a weak attempt to support the fact that smoking in the locality has declined,but does nothing to impact on the main argument concerning the reason for thedecline.

13. (D)A parallel logic question easily reducible to simple algebra. No A (no historical restorationprojects) are B (got permits this month). Some C (this firm’s projects) are A (restorations).Therefore some C (the firm’s projects) are not B (didn’t get permits). In the creditedanswer, A is “films released this season,” B is “Barker’s films,” and C is “got enthusiasticreviews,” the only difference being that answer choice (D) reverses the order in which theevidence is mentioned.

(A) No A are B. Some C are D. Therefore—um—up till now, B hasn’t been an issue for E.Ah, forget it.

(B) No A are B. Some C are not B. Therefore some D are B.

(C) No A are B. Only B are C. Therefore some A are not C. Close but no cigar.

(E) fails to give us a “No A are B” statement at all, so we can throw this one out with nofurther ado.

• When can it pay to translate Parallel Logic into algebra? When the stimulus andchoices are written in a strictly formal way, in All/Some/None terms—these are termsthat can be amenable to reduction to algebra.

LSAT PREP _____________________________________________________________ LSAT Test XVII Explained: Section III

58 © K A P L A N

14. (E)Again, we seek a necessary assumption to hold an argument together. First we get a claim,followed by the phrase “whether or not this hypothesis is correct . . .”, which stronglyimplies that the initial sentence is largely irrelevant. And indeed, the real argument beginswith the word “most”—most art criticism is directed at pieces that critics don’t like. Theconclusion follows, signaled by “hence”: most art criticism is devoted to work that isn’tgreat. This argument seems suspect on the face of it simply because the author has failed todemonstrate the necessary connection between these two statements. Just because criticsdon’t like a piece of art doesn’t necessarily mean that that piece of art isn’t a great work ofart. But this conclusion would be valid if we make the connection via the assumption statedin (E), which links great works of art with pieces of art that satisfies critics. Without thisconnection, the argument is incomplete. With it, the conclusion works: If most art criticismis devoted to art pieces that fail to satisfy critics, and great works of art are those pieces thatdo satisfy all critics, then it makes sense to conclude that most criticism is devoted toworks other than great works of art.

(A) changes the scope of the argument with a new term, “intensely,” and does nothing tobridge the gap between art that fails to satisfy and great works of art.

(B) The issue is which art works are selected for criticism by the critics The question ofwhether art works that are truly satisfying to critics are difficult for critics to find has nonecessary bearing on this issue.

(C) is irrelevant. The power of critics to publicize works of art is not an issue in theargument and doesn’t provide the connection necessary to sustain the conclusion.

(D) is out of the scope. The argument in no way hinges on the issue of when works of artare recognized.

• Formal logic statements—signaled by words like “most,” “some,” “all,” “none,” etc.—are usually buried within casual-sounding arguments on the LR sections.Sometimes, as in this case, your job will be to recognize how two or more of thesestatements can be tied together logically with the addition of an interveningassumption.

LSAT PREP _____________________________________________________________ LSAT Test XVII Explained: Section III

© K A P L A N 59

15. (B)An initial study of the question stem reveals your need to decide what kind of statementwould be countered by the given evidence, which basically states that two differentcategories of infant have similar “precursors of speaking”: Hearing kids with hearingparents babble audibly, while deaf kids with deaf parents “babble” in signs. Havingnoticed that, you probably could have pre-phrased nothing more complex than Hearingand deaf kids develop speech differently and come up with (B). The author’s argument—thatboth categories of infant start to develop speech in much the same way—certainly doescounter any claim that a characteristic allegedly necessary for language development isone that only hearing kids possess. According to the author, the deaf kids don’t possess amature vocal tract and are developing language mighty well, thank you.(A)’s topic is “What are the simplest words in a language?,” something our author nevertakes up. Also, who says that “babbling” = “names of persons or things”? What a bogusanswer choice—no offense if you picked it, but still....

(C) All of the evidence is about how babies babble with adults present, so (C)’s hypothesis isoutside our author’s scope.

(D) We don’t get enough information about the babbling of either hearing or deaf children,to counter (D)’s claim that babies are unaware of how purposive their babbling is. Andanyway, it’s the alleged deaf/hearing distinction, and not babies’ self-awareness, that ourauthor is interested in.

(E) Given that the question asks “Which would the argument counter?,” we might haveexpected a choice to provide the exact opposite, and this is it. (E) is supported by thestimulus text.

• It stands to reason: When the right answer counters an argument, the wrong choicesmust either support that argument (E) or be irrelevant to it (A), (C), (D). This kind oftest knowledge, once mastered, can help you avoid mistakes and wasted time.

• Scope, scope, scope. Don’t lose sight of how every author narrows down a broadtopic. All five choices here deal with babies and language, but only (B) narrows thescope to the “deaf vs. hearing” issue.

16. (A)Reading the stem first tells us that the reasoning is flawed, so you should have been on thelookout for the reasoning error right out of the gate. The individual pieces, the elementsthat make up the library, were all copied and are therefore unoriginal. Does it follow, then,that the completed library cannot be considered original? Not if there’s something else thatconfers originality on such a design, such as the creative process involved in choosing thepieces, or in arranging them to form a new coherent whole. In other words, something canbe true of the whole without being true of any of the parts, and so the author makes theerror described in (A).

LSAT PREP _____________________________________________________________ LSAT Test XVII Explained: Section III

60 © K A P L A N

If it’s still unclear, try this analogous example: Assume there are four musicians . . . oh, let’scall them John, Paul, George and Ringo. If none of them is considered a spectacularmusician individually, does it follow that if you put them together, the result must beunspectacular as well? Nope; it’s the same issue as the one in the stimulus—we can’tlogically assume that something true of parts of a whole must be true of the whole itself.

(B) The author does not claim that the specified features are similar in kind to the designof a library in general, and so there is no such generalization.

(C) There’s no “unknown instance of a phenomenon” mentioned, so we can stop there.

(D) Nope; there is no false “combination of alternatives” made up of alternatives thatcould be true separately. If you chose this, you have to defend it—what are thealternatives? How are they true individually, and not in combination? If you can’t answerthese questions, you can’t select this choice.

(E) It’s possible to make conclusions of fact based on reports of aesthetic preferences, andso (E) isn’t a reasoning error at all. In any case, there are no such reports in the stimulus.

• Always read the question stem first, and use it to help you attack the stimulus. Italways helps to know what you’re looking for before you begin your initial reading.

• Pay attention to the specific words found in LR answer choices—often, you caneliminate choices quickly based on a word that simply doesn’t fit the situation. Is alibrary design a “phenomenon”? Are there “alternatives” mentioned in the stimulusthat stand alone but crash and burn when placed together? Clearly not; (C) and (D)should be axed quickly just for including these words.

• Pre-phrase your answer whenever you can. Extensive evaluation of the answerchoices always takes longer than scanning for the correct response.

17. (B)How fortunate for us; two questions based on a single explanation of a phenomenon, one toweaken that explanation, one to strengthen it. The phenomenon in question is thecommercial resurgence of the fantasy genre in North America, specifically the increase infantasy-fiction books for adults. The explanation, given by booksellers, is that the recentboom in this genre is due to favorable reviews given to these works by book reviewers. It’snot too difficult to pre-phrase a weakener for this explanation—if the buyers of such booksdon’t read, care about, or base purchasing decisions on such reviews, then this argument isa whole lot of hot air. And that’s exactly what we get in (B).

(A) is generally irrelevant, because it leaves out the purchasers of books altogether. Whatpublishers think will receive good reviews is too wishy-washy to have any affect on theargument, no less weaken it.

(C) The fact that booksellers are aware of the content of the reviews is also irrelevant; it hasno bearing at all on the affect the reviews have on the purchasing public, which is, after all,what the booksellers’ argument is about.

LSAT PREP _____________________________________________________________ LSAT Test XVII Explained: Section III

© K A P L A N 61

(D) is even further removed from the crux of the argument; it totally ignores the main issueof why this type of book has become so popular.

(E) is, at best, neutral. At worst, it’s au-contraire —if people pay attention to the reviews, thiswould provide even another reason (historical) for people to get into this genre.

• Take whatever gifts the testmakers throw your way. The question stem out-and-outtells you what you’re trying to weaken—the booksellers’ explanation. So it’s not as ifyou even have to search for the conclusion; the phrase “some booksellers say that . ..” in the last sentence should have jumped off the page at you.

• Don’t be surprised at the appearance of really simple questions; not every one is alogical brain-teaser. Here, the booksellers think that favorable reviews areresponsible for the upsurge in the fantasy market. What would weaken this claim?Someone else saying “nope, not true. The readers don’t care about that.” And that’schoice (B).

18. (E)Having thought through the weakener for 17, all we have to do is reverse our thinking tohaul in this point: To strengthen the argument, we need a choice that says that bookpurchasing behavior is influenced by reviews of the kind described in the stimulus. Andthat sentiment appears in (E): As a result of critic’s favorable reviews, adult book buyersbegan to see the fantasy-fiction genre as a viable option for their reading pleasure.

(A) We have no idea how complex or simple the books in this genre are, and thereforehave no way of knowing how the general reading level of the public affects this argument.

(B), (C), and (D) all may contribute in their own ways to a general explanation of the recentpopularity of fantasy-fiction books, but none offer any support for the booksellers’ specificargument—that the popularity of these books is due in some way to favorable reviews.

• Don’t lose sight of the facet of the argument you’re attempting to strengthen orweaken. The booksellers’ argument in this case is not simply that fantasy-fictionbooks have become popular; they feel that they became popular due to favorableattention from reviewers. If the argument simply concerned the “what” and not the“why” of the matter, then (B), (C), and (D) would be viable choices as well.

• Once again, try not to miss out on double question stimuli. It just seems like forevery double-question stimulus on this test, the second one was a real breeze afterworking through the first. If you find yourself running out of time, and you thinkyou might not get to every question on the section, you may consider flipping thepages in search of a two-for-one deal.

LSAT PREP _____________________________________________________________ LSAT Test XVII Explained: Section III

62 © K A P L A N

19. (E)Both the stimulus and the credited choice provide a chain of superlatives: The Tyler house(the Oakland roses) is the most famous in the historic district (most beautiful in the garden),and the historic district (the garden) is the most famous in the city (most beautiful in the city) ,so the Tyler house (the roses) is the most famous in the city (most beautiful in the city). All ofthe choices come more or less close, but only (E) strictly follows this model. And of courseboth the stimulus and (E) are profoundly flawed: It’s quite possible that the fame of someother house in the city eclipses that of the Tyler house, or that flowers in some other citygarden are more beautiful than the Oakland roses. The only evidence is the context of thehouse and the roses; we can’t reason from the relative fame (or beauty) of the house’s (orroses’) surroundings, to the individual itself.

(A) and (C) offer incontrovertible logic. Tallness and age—unlike fame or beauty—are notrelative. If Mt. Williams is the tallest peak along the coast, and the region’s tallest peaks arealong the coast, then Mt. Williams is the region’s tallest. How could any mountain,elsewhere in the region, be taller? And if Susan Coleman is the oldest kid in her family,and all three Coleman kids are older than all other kids in the building, how could any kidin the building be older than Susan?

(B) Whew—more tobacco smoking in Greene County than anywhere else?! Anyhow, (B)’sreference to “likelihood” knocks this one out; no such reference to the likelihood ofanything happening appears in the stimulus.

(D) comes very close, but the middle piece of evidence goes awry. If it read “Since the fishstores in the harbor area have the most exotic fish of any stores in the city....,” (D) would beparallel: What links the terms in the stimulus is the concept of fame (‘exoticness’ here). Butthe number of fish stores that the middle evidence mentions has no parallel in the stimulus.

• Be sure to examine all five choices, especially when the question seems tricky andthe answer choices seem close to each other. Many examinees in December, 1995jumped on (B) and (D) here, and never even got to the credited answer.

20. (A)Compare the first two lines of Morton’s speech (his conclusion) to the last two (theevidence, signaled by the Keyword “since”). All he does is repeat his claim, that successrequires a college degree, but he provides no independent evidence for same. We’ve seenthis kind of circular logic before, most recently in the first LR section, Q. 2.

(B) An example of mistaking correlation for cause-and-effect: “All of the students at DrakeMiddle School wear uniforms, and they’re all well-behaved; it must be the uniforms thatmake them behave.” It’s a common error, but it’s not committed here.

(C) The argument, such as it is, is general in nature. That Morton ends by mentioning “aperson” doesn’t change things—he’s not speaking about one specific person.(D) Morton does consider the counterexamples: He looks at them and finds them wanting;they are only “apparent” counterexamples.

LSAT PREP _____________________________________________________________ LSAT Test XVII Explained: Section III

© K A P L A N 63

(E) Morton’s evidence and conclusion are his own beliefs, and do not imply that which“most people believe” in the slightest.

• If you can’t pre-phrase the answer to a Logical Flaw question, proceed very carefullyas you examine the five choices. Remember that one and only one answer fits thestimulus precisely; the other four must be off, and probably way off. Don’t settle foranything less than a solid choice, and if you don’t spot one, circle the question in thetest booklet and come back later.

21. (A)Another assumption question: In this one, the author concludes that the adaptations ofearly land animals must have developed quickly after the animals left the water andstarted to live on land. As evidence, the author notes that neither aquatic nor amphibiousanimals have these adaptations, and all the earliest known fossils have them. Therefore, ifthe gap between the time animals first started living on land and the time of the earliestfossils was short enough, then the adaptations would have had to develop relativelyquickly. If, on the other hand, the earliest known fossils lived a long time after animalsstarted living on land, then the adaptations would have had a longer time to develop, andthe author’s conclusion would be in jeopardy.

(B) has no bearing on the argument. The author draws a conclusion based on existingfossils; the percentage of animals from the late Silurian period that turned up as fossilsdoesn’t fit in any way between the fossil evidence and the conclusion drawn from it.

(C) is simply out of the scope; the argument concerns adaptations in animals, not plants.

(D) The argument discusses adaptations in the evolution of animals from water to land. Itneed not rely on evidence that this evolution can go in the reverse direction as well, fromland to water. Even if some present day water animals did descend from land animals, theauthor’s argument remains unaffected.

(E) If there were amphibious animals at the time, they couldn’t have exhibited theadaptations; the passage tells us than neither aquatic nor amphibious animals had them.

• Paraphrase and pre-phrase! The stimulus will give you enough information toanswer the question, but rarely in the form that is easiest to understand. The bestapproach is to put the argument in your own words. Once you have a strong sense ofthe argument, think of what the answer should look like, and then searchaggressively among the choices.

LSAT PREP _____________________________________________________________ LSAT Test XVII Explained: Section III

64 © K A P L A N

22. (E)The reasoning to be criticized here is the author’s claim that Jerome is being disingenuouswhen he, Jerome, cites cost as the reason for declining Melvin’s vacation invitation. Whymust Jerome be lying, or perhaps fudging? Because, says the author, Jerome cites costevery time Melvin tenders the invite. Well come on, so what? Maybe the combined cost ofthe trip plus the lost wages is always the one and only reason Jerome says no to theseimpromptu jaunts. The author simply fails to take Jerome at face value, as (E) points out.Once again, the imputation of bad motives to a person without proper evidence is shownto be faulty.

(A) No attack on Melvin is being planned, so none need be “forestalled.”

(B) Melvin’s the one making the offers. It’s Jerome’s refusals that are the topic of theargument.

(C) If Jerome does prefer vacations planned in advance but simply fails to mention that toMelvin, that actually tends to support the author’s suspicion that Jerome is fudging things.

(D) Tempting, perhaps, but the author doesn’t say that cost can’t be Jerome’s only reason,she says it can’t be the real reason. It’s the adequacy of the explanation that’s in question,which brings us back to (E).

• It’s logically wrong to impute bad motives to someone without legitimate evidence.So many of us commit that error in everyday life that the LSAT is always interestedin testing it—usually in wrong answers, but here as the focus of the question.

23. (C)What a stem! First recognize that we’re looking to justify Jamie over Arnold. Thenparaphrase the rest of it, and recognize that the issue is: When does an airline have a moralobligation to reimburse a bumped passenger? The stem gives us the beginning of agoverning principle, and we have to find the appropriate finish.

Arnold believes that the answer to the question “When should the airline pay?” is “In mycase, dammit,” because he was bumped from a flight and missed a meeting. Notice whatArnold casually tosses in: Of course, the flight I was bumped from was canceled. Jamie’sresponse is essentially, Gimme a break: You can’t use that missed meeting as an excuse,because you would have missed it even if you had boarded the first (canceled) flight.

Now: The stem begins the principle in a positive sense: “An airline is morally obligated tocompensate” etc. But of course Jamie doesn’t support Arnold’s claim. So the rightanswer—follow this, now—will have to be a condition that is necessary for compensation,but that Arnold’s situation fails to meet! And that’s what we get in (C). Arnold wasn’t forcedto take a later flight solely because of the overbooking. Had the first flight not beenoverbooked, Arnold couldn’t have taken off anyway; and thus this particular instance ofbumping need not be compensated.

(A), if attached to the stem principle, might seem on a quick first read to support Arnold’sview. But overbooking was not “the only reason [Arnold was] forced to take a later flight.”

LSAT PREP _____________________________________________________________ LSAT Test XVII Explained: Section III

© K A P L A N 65

The cancellation of the earlier flight was another reason, and certainly the more compellingreason. So Arnold’s case doesn’t fit this principle—we can’t use it to draw a conclusion.

(B) provides a condition necessary for compensation that Arnold’s case meets: There was areason for his later flight other than a weather cancellation. So (B) supports Arnold’s claim.

(D) and (E) Both choices can be tossed the moment you read “Even if.” An “even if” clausedefines a condition that is not to be considered in applying a principle, as for instance: Youshould get up and take out the garbage for your mom even if you’re watching TV at the time. Any“even if” clause applied to the stem principle will work out in Arnold’s favor, because itwill lead his being compensated. Since we want to support Jamie over Arnold, bothchoices are dead wrong.

• When a stem is highly complex—as this one is, and also Q. 24’s, below—rememberKaplan’s advice for complex stems, rules, and text in other sections: Don’t try to“swallow it all in one gulp.” Take it one phrase at a time, in manageable doses;paraphrase carefully. And be sure that you get to all the other, easier questions first,so that you don’t sacrifice any quick points for a tougher one.

24. (E)This one turns out to be a Paradox question even though it’s not formally announced assuch. It does seem a bit paradoxical that Korva’s population went up and yet its percentageof federal revenues declined; after all, revenue sharing is pegged to population, isn’t it?Well, yes and no. As the author describes it, the $ is divvied out in terms of percentage oftotal population, not sheer number. Korva can easily have gotten a smaller % of the cashdespite its population increase, if Mitro or Guadar or both also saw a population increase,and by a greater proportion. Which is what (E) states. To test this, use the Kaplan DenialTest: If (E) is false—if neither of the other regions saw a greater percent increase inpopulation than Korva—then the paradox deepens.

(A) Even if Korva does have the smallest of the three regions’ populations, its populationlast year did increase, and without knowing what happened with the population of theother two regions we have the paradox that Korva’s money decreased while its populationwent up.

(B) provides information about Korva’s change only. The issue is why Korva got a smallerpiece of the pie than did Mitro and Guadar, and choice (B) doesn’t address that any morethan (A).

(C) could have been true, but need not. The smaller % of cash awarded to Korva can beexplained so long as either of the other two regions saw a greater percentage populationincrease. That’s what correct choice (E) is all about. One of the two regions, Mitro orGuadar, could have had a stable percentage of total population, or even a drop.

(D) Contrary to choice (D), Korva could have had the greatest numerical populationincrease, and still the paradox would remain. The revenue dollars are pegged to percent ofincrease.

LSAT PREP _____________________________________________________________ LSAT Test XVII Explained: Section III

66 © K A P L A N

• The Kaplan Denial Test works nicely on Paradox questions as well as on Inferenceand Assumption questions. In all cases, the right answer—if denied—mustcontradict the text. (A wrong choice, such as answer choice (D), can be denied withno effect on the argument; see above.) Try denying the choices to see whether youhave picked a winner.

25. (E)More fossils, and more beetles, too! The last time they were in the paradox of question 2.No paradox here, just another assumption. This stimulus describes a pretty niftyprocedure: The research team first establishes the temperature tolerances of currentexisting beetles, and then finds and dates fossils of the same beetle species. The purpose isto establish maximum summer temperatures for particular years in Britain by matchingthe tolerances of existing beetles to the places and periods determined by the fossils. If acurrent species X beetle can withstand a maximum of 90 degrees, and a fossil of a species Xbeetle is traced to a county in Britain, 15,000 years ago, the researchers would conclude thatthe maximum temperature in that county 15,000 years ago was 90 degrees. But theresearchers are assuming that the tolerances of the beetle species studied haven’t changedin the interim time period. The argument therefore needs the assumption in (E)—if thetolerances have changed, the whole connection is shot to hell and the researchers can nolonger use their findings as a reliable indicator of temperature conditions in bygone years.

(A) is irrelevant. Even if beetles functioned better in cold weather, the research team couldstill use their presence to set maximum temperatures as long as their tolerances have notchanged.

(B) Why would this be necessary to the argument? The fossils of various species found inthe same place could belong to the same time period without causing a problem for theresearchers.

(C) is an irrelevant comparison. Presumably, the process of dating beetle fossils needs to bereliable, but it need not be more reliable that the process of dating other organisms in orderfor this argument to remain valid.

(D) is long and wrong. The highest actual temperature couldn’t have been higher than thetemperature tolerances for any one of the beetles known to have lived there. If the highestactual temperature was an average of the highs for each beetle, this would necessitate thatit was higher than some beetles’ tolerance (unless all the tolerances were exactly equal),which, according to the setup of the study, is impossible.

• Always try to pre-phrase your answer. Some questions are simply not amenable topre-phrasing, but others, like this one involving a fairly blatant assumption, are.When you have an inkling of what you’re looking for, you can save time byscanning the choices aggressively, looking for the one that matches your prediction.In this one, you may have been able to skip past the nightmarish (D) if your scanrevealed that (E) was the ticket.

© K A P L A N 67

SECTION IV:

READING COMPREHENSION

LSAT PREP _____________________________________________________________ LSAT Test XVII Explained: Section IV

68 © K A P L A N

Passage 1—Zora Neale Hurston(Q. 1-8)

Topic and Scope: Zora Neale Hurston; specifically, different assessments of her novel,Their Eyes Were Watching God.

Purpose and Main Idea: The author’s purpose is to explain why Hurston’s novel has onlyrecently received widespread praise, her main idea being simply that the development ofnew strands of literary criticism accounts for this turnaround.

Paragraph Structure: ¶1 introduces the passage’s basic question: Why was such animportant and eventually influential novel dismissed when it first appeared? ¶2 says thatthe novel was misunderstood and condemned, especially in the Black literary community,because it was out of step with the Black “protest fiction” of the time. ¶3 explains that therise of feminist and Afrocentric literary criticism has been behind the new appreciation ofHurston’s novel.

The Big Picture:

• Passages like this one—where topic, scope, and purpose become clear early on—are agood place to begin work on the Reading Comprehension section on test day.Passages that begin with a lot of unfocused details are best left for later in thesection.

• This passage features a classic structure that you’ll surely see on test day. A“problem” is presented up front; and the rest of the passage is devoted to probingand “solving” this problem.

• Book review passages often mention different authors and different works. Keepthem straight in your mind—the questions will probably test to see that you’re awareof the differences between/among authors and books.

The Questions:

1. (E)Lines 15-18 say that Wilson’s work, “unlike” Hurston’s, was totally ignored upon itspublication. But, like Hurston’s work, it has recently been the object of literary inquiry.

(A) The passage doesn’t say when Wilson’s book was published, so we can’t conclude thatit was written at the same time as Hurston’s novel. Moreover, Wilson's work didn’t receiveany critical attention.

(B) According to the passage (lines 1-5), it’s Hurston’s novel that has heavily influenced thewriting of later Black women.

LSAT PREP _____________________________________________________________ LSAT Test XVII Explained: Section IV

© K A P L A N 69

(C) This passage is mostly concerned with the literary establishment’s tardy appreciationof Wilson’s novel, not with the public, but even the few references to the public suggestthat (C) is off. Readers were put off (lines 36-38), “and the novel went quietly out of print”(line 40)—not signs of wide readership.

(D) Lines 15-18 make it clear that Wilson’s novel was published.

• Although this is an inference question, you don’t really have to infer anything. Youjust have to read lines 15-18 carefully. On test day, you’ll find that many inferencequestions don’t require you to do more than read carefully.

• Harriet Wilson is mentioned only once (in lines 15-17). Read the lines around such areference carefully, for that’s where you’re most likely to find the correct answer.

• A one-word Keyword signal such as “unlike,” line 15, can have the power to unlockan entire question. Find and use the Keywords you're given!

2. (B)¶2 mentions that Hurston’s book went out of print because it wasn't understood by muchof the Black literary establishment and readership. (It was Harriet Wilson’s novel, notHurston’s, that was relegated to obscurity because of critical neglect.)

(A) Au contraire. Hurston’s novel received a lot of flak from reviewers when it appeared.

(C) Afrocentric interpretations of Hurston’s novel, ¶3 reveals, are a more recentdevelopment.

(D) ¶s 2 and 3 make it clear that Hurston’s work doesn’t belong to the genre known as“protest fiction.”

(E) The passage says that the book received mixed reviews from White and Black critics.There’s not enough information to conclude that “most” reviewers gave it a less thanpositive reception.

• Question stems sometimes don’t point you to a specific part of the passage. Whenthis happens, your best approach is to go through the choices one-by-one, withouttrying to pre-phrase an answer.

3. (C)This choice captures the author’s topic (Hurston’s novel); scope (earlier versus laterassessments of her novel); and purpose (to explain why Hurston’s novel has recently wonacclaim).

(A) focuses on a detail in ¶2.(B) According to the author, Afrocentric literary critics have also helped to bringHurston’s novel to prominence. Besides, this passage also discusses why Hurston’s novelwasn’t well-received initially—an aspect of the passage that (B) doesn’t address.

LSAT PREP _____________________________________________________________ LSAT Test XVII Explained: Section IV

70 © K A P L A N

(D) Hurston’s novel was trashed by some early literary critics precisely because it wasn’t“protest fiction.”

(E) focuses on a detail in ¶3.

• The correct answer to a global question must be broad enough to encompass thecontents of the entire passage. Watch out for choices that focus on individual detailsor ¶s.

4. (E)According to lines 52-56, the recent rise of Afrocentric literary analysis has brought to lightBlack folklore traditions as represented in literature.

(A) distorts a detail in ¶3. Indeed, the only thing we’re told about Gates is that he haswritten perceptively about Hurston’s novel.

(B) and (D) are beyond the scope of the passage, which is about literary criticism ofHurston’s novel. We aren’t told much about Black novelists as a group. For all we know,Black novelists writing before or during the 1940s did use folklore in their works.

(C) Again, we don’t know what other Black authors were doing; so, we can’t conclude thatHurston was the first to incorporate Black folklore into a novel.

• Don’t rely on your memory to answer detail questions. Go back to the passage andreread to make sure that you’ve got the details straight.

• During your work on the questions, keep remembering the purpose of eachparagraph—the result of the “roadmap” you assembled during the reading. Here,¶4's scope of “recent developments” should frame your consideration of what'sgoing on in lines 52-56.

LSAT PREP _____________________________________________________________ LSAT Test XVII Explained: Section IV

© K A P L A N 71

5. (C)¶2 strongly implies that Wright’s Native Son fell into the genre of “protest fiction,” whichexplored the issue of racism in the United States. In other words, whereas Hurston’s novelconcentrated on the life of one individual in the Black community (lines 39-40), Wright’snovel focused on the entire community.

(A) Since Wright’s book fit into the dominant genre of the era, it’s highly unlikely that itgot a worse reception than Hurston’s novel, especially since we’re told that Wright's novelwas “much acclaimed.”(B) Au contraire. Wright’s novel is far more typical of mid-century Black writing thanHurston’s.

(D) We can’t say that Wright’s book has got more folklore in it than Hurston’s, because thepassage doesn’t tell us exactly what’s in his book. Except for the one reference, Native Son isoutside the scope of the passage.

(E) No comparison is possible between Wright’s book and Hurston’s on the issue offeminist and Afrocentric attention, because the passage doesn’t provide us with feministand Afrocentric comments on Wright’s novel.

• Many inference questions will contain wrong choices that fall outside the scope ofthe text. Here, choices (A), (D), and (E) are all outside the scope.

• Wrong answers often blow up the importance of a detail. Notice how, in thispassage, Native Son is only dragged in as a single simple point of comparisonwith These Eyes..., yet Q. 6's wrong choices all imply that the passage has much moreto say about Wright's novel than that.

6. (B)Lines 29-33 say that Black writers of the 1940s believed that Black literature should fall intothe “protest fiction” genre—that is, they believed that it should tackle the issue of racismagainst Blacks. The novel outlined in choice (B) has just such a focus.

(A), (C), (D), and (E) None of the novels in these choices focuses on racial discrimination.(D)’s “unjust working conditions” is not the same as racial discrimination.

• One or two questions on test day may ask you to apply information in the passage toan “outside scenario.” If you don’t see the idea that the question’s testing right away,move on to another question. There’s no sense wasting time; the answer to this sortof question won’t be found in the passage. Come back to it later if you’ve got time.

LSAT PREP _____________________________________________________________ LSAT Test XVII Explained: Section IV

72 © K A P L A N

7. (C)In ¶2, the author notes that some mid-century reviewers dismissed Hurston’s book becauseit doesn’t deal with racism, the issue that interested them. In ¶3, the author notes that laterreviewers, influenced by feminism or Afrocentrism, applauded the book because it dealswith issues of concern to feminists and Blacks. Hence, the author is likely to agree with thenotion that literary critics’ “ideological perspectives and assumptions about the purposeof art” color their analysis of art.

(A) and (D) The author would probably disagree with these statements. After all, shelargely attributes the early demise, as well as the later revival, of Hurston’s novel to theefforts of literary critics.

(B) The author doesn’t discuss any “experimental” works of fiction, whatever that termmeans. Thus, there’s no telling what she’d have to say about how literary critics initiallyview them.

(E) is too strong. True, the author notes that literary critics make much of the ideologicalpremises of a book; but there’s nothing in the passage to indicate that she thinks that theyhone in on this aspect of a work above all else.

• Most passages will have one or two challenging questions. Don’t let them bog youdown. Remember, you don’t get extra credit for answering hard questions correctly:All Q's are worth the same. Do all of the easy questions first; then come back to thetougher ones if you’ve got the time.

8. (B)The passage is mainly about the reassessment of Hurston’s novel that grew out of newforms of literary criticism.

(A) What misconception?

(C) distorts the contents of the passage. True, the author discusses early and more recentcritiques of Hurston’s novel; but she doesn’t attempt to “reconcile” them.

(D) What conventional approach?

(E) What new discovery?

• If you got Question #3, you should’ve gotten this one. When two questions probe thesame issue, use the answer to the easier one to help you get the tougher one.

LSAT PREP _____________________________________________________________ LSAT Test XVII Explained: Section IV

© K A P L A N 73

Passage 2—”Hard” Cases in Law(Q. 9-15)

Topic and Scope: The concept of “hard” (tough; controversial) legal cases; specifically, towhat extent such cases are “legally determinate,” defined by the author as “decidableunder existing law.”

Purpose and Main Idea: The author’s purpose is to compare two legal thinkers' views ofthe law as a means of evaluating when and how controversial cases can be decided underexisting law. By the end, certainly in the final sentence, we see that the author finds merit inboth views but, in the main, finds Hart's conception of the open-endedness of legalterminology to be both persuasive and useful.

Paragraph Structure: ¶1 presents some fundamental terminology and definitions—”hard”cases and “determinate”—as it lays out the passage’s fundamental issue: How can hardcases be decided? ¶2 belongs to Hart: his conception of law as legal rules with “open-textured” general terminology; an extended example; and the suggestion that some casesneed to be decided on moral or political, rather than strictly legal, grounds because theirterminology isn't specific or determinate enough. Dworkin is the focus of ¶3: To him thelaw isn't just Hart's rules, but also includes principles, and the two work in tandem to render“legal indeterminacy”—the deciding of cases on other than legal grounds—a non-issue. ¶4,as noted above, is comfortable with both concepts—both rules and principles—as theauthor forges a middle ground, deciding finally that there do exist difficult cases and abranch of law where things are simply not cut-and-dried, and judges must exercise somediscretion.

The Big Picture:

• This passage is a strong candidate for being left until last: It's dense and difficultfrom the first few lines. Most students find this kind of windy text to be moremanageable if it's approached after a lot of easier points are under one's belt.

• Don't be nervous when a passage seems to involve a lot of intricate field-specificjargon, as this passage does. If the jargon important enough, then it will be clearlydefined.

• It is quite possible that you successfully attacked all or a majority of this passage's 7questions without really knowing what the author was talking about. Don’t worry,because that is quite common. The beauty of LSAT Reading Comprehension is thatif we get the gist of what's being discussed in a passage, we get a strong handle onthe questions posed.

LSAT PREP _____________________________________________________________ LSAT Test XVII Explained: Section IV

74 © K A P L A N

The Questions:

9. (C)Dworkin's concept is legal principles; Hart's is open-textured legal rules; ¶4 indicates theauthor's level of comfort with both; and (C) sums it all up and gets the topic and scoperight, to boot.

(A) begs the question of the topic and scope—the determinacy of “hard” cases—and comesdown squarely on the side of the need for resolving all cases through law alone, somethingthe author rejects.

(B) is the choice for those who grab ¶1 and not much else. The definition of “hard” cases isa jumping-off point for the real issue of how such cases are to be decided, and (B) mentionsnone of that.

(D)'s negative judgment of Dworkin's concept of legal principles is not supported by theauthor—indeed, the author embraces the concept in ¶4.

(E) is a pretty radical distortion of this passage's content and tone. Hart's definition of legalterminology as “open textured” implies (quite sensibly) that language has some inherentflexibility of meaning—even legal language—and that judges need to exercise somejudgment as a result; but all of that is a far cry from accusations of “inherent inconsistency”and “defects and gaps in the law.”

• As long as you recognize the “middle ground” between Hart and Dworkin that theauthor is trying to forge in ¶4, you should be able to reject the four uncreditedchoices without straining for full comprehension of the text.

• We must always be conscious of topic and scope, and never more so than in globalquestions, where dozens of wrong choices can be put by the wayside simplybecause they deviate from the author's broadest terms of argument.

10. (C)This choice nicely paraphrases lines 40-41.

(A) sounds like a plausible definition of the term “legal principles” on its face, but it hasnothing to do with Dworkin or, indeed, with anything else in the passage.

(B) Legal principles, as defined by Dworkin, don’t explain legal rules: They coexist withthem, within the body of established law (see lines 35-37).

(D) Dworkin believes that law exists outside of legal rules.

(E) Dworkin seems to be opposed to the notion that some cases need judicial discretion(lines 42-45), so he would not coin the term “legal principles” to mean cases that requiredsame.

LSAT PREP _____________________________________________________________ LSAT Test XVII Explained: Section IV

© K A P L A N 75

• You're pointed to ¶2 by this stem, but again, you don't necessarily need chapter-and-verse understanding of ¶2. When you're asked for a definition, as you are here,ignore distractions and just concentrate on the definition that the author provides.

11. (D)The answer comes from the final sentence—as well as the realization that rules are Hart'sconcept, principles are Dworkin's, and the author is trying to effect a synthesis of the twoin the final paragraph. The “penumbra of both rules and principles,” of course, is (lines 15-16) the place where things are not cut-and-dried and determinate.

(A) The author never posits which—rules or principles—is appealed to more often in hardcases.

(B) “Official recognition,” whatever that means, has nothing to do with this “think piece”on legal determinacy, the author's effort to work out a theory of one part of the law.

(C) Far from arguing that rules have been superseded by principles, the author believesthat both exist in the law.

(E) No, the author hints strongly that those cases “in the penumbra of both rules andprinciples” (lines 56-57) will, in fact, require judges' discretion for their decidability.

• Even when you can't quite pinpoint where in the passage an answer is to be found,you can be bold in rejecting choices (such as answer choice (C) and answer choice(E) here) that are wildly at odds with the author's point of view.

12. (D)This choice sums up lines 18-22.

(A) and (B) are off because the word “vehicle” is a term, not a rule or principle in and ofitself.

(C) and (E) Au contraire, “vehicle” was chosen precisely because it does not necessarilyhave one settled meaning (C), because it's Hart's concept in action (E).

• Don't strain too hard on “purpose of a detail” questions. Try to take them at facevalue.

13. (B)This answer comes directly out of lines 8-11, which tie Hart's concept to the standard lawand describe Hart's ideas as “clear” and “persuasive.”

(A) “Clear” and “persuasive” don't imply that a work or theory covers a topic“exhaustively.” No evidence for this one.

LSAT PREP _____________________________________________________________ LSAT Test XVII Explained: Section IV

76 © K A P L A N

(C) and (D) are each half-right, half-wrong. Interesting, sure; plausible, o.k.; but“impractical” or “unwieldy”? It's unlikely that a theoretical think piece like this onewould traffic much in practicality anyway; but if anything, the author does seem to seepractical and workable (as opposed to “unwieldy”) value in Hart's concept (see ¶4).

(E) Hopelessly wrong. No matter how far back in history Hart's work dates—and of coursewe get no info to that effect—the author seems to find it relevant and well worth studying.

• Don't just answer tone questions in a seat-of-the-pants way. Scour the passage forexplicit textual support for your choice.

14. (B)Some examinees, thrown by all of the heavy-duty terminology in ¶1, failed to see that“determinate” is clearly defined by lines 6-7 and paraphrased in (B). Very straightforward.A demonstration of how one's frame of mind can make or break a particular LSATquestion.

(A) Penalties—sentences, punishments, etc.—are way beyond the scope of thisinvestigation.

(C) The concept of judicial discretion doesn't come into the passage until much later, andthen only in passing. This choice picks up on some terminology from the previoussentence (lines 1-3) but is otherwise incoherent.

(D) defines “indeterminate” pretty well.

(E) No reference to codified procedures can be found anywhere near line 6.

• Frame of mind is so important. Remember that if you're struggling with a passage'sdense ideas or difficult jargon, so is everyone else; and trust that a general sense ofthe issues involved should be enough for at least the lion's share of the questions.

• Remember, too, that even difficult passages come equipped with less thanchallenging questions. Like this one.

LSAT PREP _____________________________________________________________ LSAT Test XVII Explained: Section IV

© K A P L A N 77

15. (E)This choice as written basically just sums up two lines, lines 46-47, but that fact shouldn'tunduly trouble you: After all, ¶4 is where the passage's individual discussions of Hart andDworkin is heading—it is, therefore, the most important ¶—and lines 46-47 do encapsulate¶4. So despite (E)'s evocation of only two lines, they are the most important lines and canstand acceptably as the author's purpose.

(A) Nonsense. The in-practice role of the legislative branch is about as far removed fromthis passage's concerns as can be.

(B) is an issue that was raised and disposed of in ¶1. The topic may be “hard” cases, but thescope moves away from their definition to their determinacy, something that (B) ignoresbut (E) exploits.

(C) paraphrases lines 38-41, but these lines aren't nearly as all-encompassing as lines 46-47.And in terms of the author's main purpose, Dworkin certainly doesn't deserve the pride ofplace that (C) assigns him.

(D) If anyone is in the “critiquing Hart” business, it's Dworkin and not our author, who istrying to synthesize the views of the two thinkers.

• When working with global questions, make a special effort to pre-phrase an answer,in order to avoid being sidetracked by tangential issues.

LSAT PREP _____________________________________________________________ LSAT Test XVII Explained: Section IV

78 © K A P L A N

Passage 3—Impose a CO2 Tax?(Q. 16-21)

Topic and Scope: A tax on CO2 emissions as a way of reducing air pollution; specifically,whether and how governments should impose such a tax.

Purpose and Main Idea: The author’s purpose is fairly clearly announced in lines 17-19,and expanded in lines 44-45—overall, he wonders, what are the issues involved inimposing that pollution tax? A flat-out conclusion that the tax is, or is not, desirable wouldact as a strong main idea, but the author falls short of providing one; he sees the upsidesand downsides of the idea, but makes no final recommendation.

Paragraph Structure: ¶1 announces the issue quite blatantly, and the rhetorical question(lines 3-7) sets the tone for the rest of the inquiry: Why impose a CO2 tax rather than anynumber of other ways to control pollution? The rest of ¶1 describes the advantages of thetax.

¶2 explores the uncertainty (announced in lines 17-19) over how high a tax would need tobe, to be effective; and then the new issue it introduces, that of cooperation betweennations, turns out to drive ¶3. That ¶ lists the difficulties in convincing all nations toimpose the tax, a topic that segues neatly into ¶4's consideration of the pros and cons ofunilateral imposition of the CO2 tax.

The Big Picture:

• Not every passage contains a “main idea.” Don't strain to find one. Some passages(like this one) simply explore the pros and cons of an issue, or lay out a narrativewith no real persuasive purpose. The best clue that a passage can be boiled down toone “big idea” is the presence of a question explicitly asking for one.

• Not every passage contains a main idea, but every passage has a purpose. Everyauthor has a solid reason for writing what she does. Explore that, first and foremost.

LSAT PREP _____________________________________________________________ LSAT Test XVII Explained: Section IV

© K A P L A N 79

The Questions:

16. (B)The details of the tax come up in ¶1, where we're told that “a carbon tax would vary withthe type of fuel” (lines 12-13). “Type” isn't immediately visible among the choices, so weneed to look to two lines earlier, where we see that the type of fuel is pegged to the amountof pollution each creates—choice (B).

(A) Tricky, because it sounds superficially plausible; but it's unsupported by anything in¶1, or anywhere else for that matter.(C) and (E) The size of the CO2-burning industries, and the number of users, are neverdiscussed. These choices seem to have been jury-rigged to fill out the set of five.

(D) Economic implications aren't mentioned until ¶2, and even then they're described asbeing uncertain, so how could the author believe that they would cause the variance in theamount of the tax?

• Your pre-phrased answer may not always appear among the choices. Instead offorcing a choice to fit, go back to the text and reread the relevant paragraph. You mayhave missed something.

17. (A)Nothing could be clearer than line 22's Keywords “for example,” indicating that what it'san example of can be found just earlier. And indeed, (A) is an easy paraphrase of lines 19-21.

(B)'s reference to “most accurate information available” is puzzling at best, and misleadingat worst: Right after presenting the example, our author (in lines 27-34) makes it clear thatthe writer's information is highly speculative in nature.

(C) No, the stats are cited approvingly.

(D) Incentives for emissions reductions are the topic of ¶1, not ¶2.

(E) Read carefully! Lines 22-27 don't show how to calculate an effective tax; they show theresults of such calculations. Exactly how the writer came up with those figures is neverdiscussed by our author.

• Never stop looking for, and relying on, Keywords.

LSAT PREP _____________________________________________________________ LSAT Test XVII Explained: Section IV

80 © K A P L A N

18. (A)A difficult question—difficult because it's based on a dense part of the passage, andbecause it's logic-based—worth skipping over in your first pass through the passage andquestions. The premise of the tax, as outlined in ¶1, is that the highest-polluting fuel wouldbe taxed the highest, an incentive to fuel users to switch to less-polluting (and, inferably,lower-taxed) fuels and to reduce energy use altogether. But if (A) is true, if the highest-taxed and highest-polluting fuel is the cheapest, then that would be a disincentive toswitch according to plan. Industry might well stick with the more damaging fuel and paythe higher tax on it, because doing so could result in net savings.

(B) The Toronto Conference doesn't come up until several lines later. Besides, if as (B)says, all fuels would be taxed higher, then that would have no effect on the conclusion inquestion.

(C) Irrelevant distinction that fails to affect the likelihood of industry substituting less-polluting fuels for the higher-taxed ones.

(D) and(E) sound so similar that some examinees may smell a rat, and suspect that one ofthem must be right and the other wrong. But whether it's gas or coal that proves to provideless CO2 than was once thought, we are still no closer to damaging the author's convictionthat industry will be moved to switch fuels after the tax is imposed.

• The LSAT shouldn't be fought as a siege. Never dig in your heels and dwell on oneparticular difficult question—especially when there may be many easier questionsjust around the bend. Circle the toughie, and go on for now.

19. (B)As noted above, this one sums up the topic and scope and the author's overall movementthrough the passage.

(A) ¶2 only.

(C) Lines 27-34 only.

(D) Answered by lines 1-3. Then what? This one misses the entire thrust of the passage.

(E) Never discussed. Too specific and technical for this kind of general-audience piece,anyway.

• This, like all other global questions, warrants reference to the entire passage as awhole. Anticipate that wrong choices will be too narrow, or irrelevant to the issues athand.

LSAT PREP _____________________________________________________________ LSAT Test XVII Explained: Section IV

© K A P L A N 81

20. (C)This one requires an understanding of the sweep of ¶4—and the whole sweep, because ifyou stop reading at line 48 you probably end up choosing wrong choice (D). Up to line 48,the author describes the benefits of a country imposing the tax on its own, but thereafterlays out the negatives. In the end, the conclusion that as a result of unilateral action,probably more CO2 would enter the atmosphere than before, strongly suggests that theauthor votes “thumbs down” to a unilaterally-imposed tax, and for the reason summed upneatly by (C).

(A) has the thrust of the discussion 180 degrees off. The rest of the world would initiallybenefit, but the taxing country would be harmed.

(B) Au contraire, other countries would benefit—at least initially.

(D) Again, a tempting choice if one goes no further than line 48.

(E) The whole idea of countries being “inspired” to do positive and idealistic things ismarkedly absent from ¶4.

• A good rule of thumb: When you think you've found your answer in the text, readon just a little bit further, to see whether an unexpected but relevant shift occursnearby.

21. (E)Look just prior to line 41 for the definition of the “free rider” effect: What we get is acountry refusing to cooperate in a group venture, and profiting rather than beingpunished for its include-me-out attitude. That's the position (E)'s shepherd is happily in,continuing to graze his sheep on fields that everyone else is eschewing in favor ofpurchased feed.

(A)'s city is trying to negotiate a good deal for itself—not exactly selfless or altruistic, butnot an individual stepping to its own beat against the wishes of others.

(B) Self-interest ($) motivates this guy, but again, he's not benefiting from his refusal to goalong with everyone else.

(C) Here's someone Doing The Right Thing despite the apathy of his neighbors.

(D) Read carefully! The homeowner's purchase of bottled water is a different approach tothe problem of contaminated groundwater from that of his neighbors and their fund, butdoes not amount to his profiting from going in a different direction.

• Analogy questions can be tricky. Get support from the text, and don't settle for achoice that seems to relate to the given situation; demand a close fit.

LSAT PREP _____________________________________________________________ LSAT Test XVII Explained: Section IV

82 © K A P L A N

Passage 4—African Drought(Q. 22-27)

Topic and Scope: Sub-Saharan West Africa’s drought; specifically, a hypothesis thatexplains why sub-Saharan West Africa has been caught up in a long and severe drought.

Purpose and Main Idea: The author’s purpose is to describe and assess a widespreadhypothesis (the “cooling” theory) about the cause of sub-Saharan West Africa’s drought;the author’s main idea is that this hypothesis is unconvincing.

Paragraph Structure: ¶1 sets out the basic hypothesis: A cooling of the NorthernHemisphere has caused the drought. ¶s 1 and 2 explain the reasoning behind thishypothesis. Essentially, atmospheric dust has reflected sunlight away from the ground,lowering the Northern Hemisphere’s air temperature, which, in turn, has affected windpatterns, ultimately leading to less rainfall in sub-Saharan West Africa. Moreover, thisdetrimental development, proponents of the hypothesis believe, might be long term.

¶3 shifts gears from description to opinion. In this ¶, the author basically says that thehypothesis is refuted by the meteorological facts.

The Big Picture:

• This passage contains a mass of details. Don’t worry about assimilating all of themduring a first read through. Instead, get a sense of each ¶’s purpose. This way you’llknow instantly where to find the details that you may need.

• It’s sometimes easier to grasp a complicated scientific process if you create a pictureof it in your mind. Passages like this one that have long descriptions of a process arevery susceptible to this strategy. Even non-meteorologists and non-geographers canget an adequate sense of the cooling hypothesis if s/he pictures the process mentally.

• Tough passages often have easy questions. Don’t blow off a tough passage becauseyou think that you won’t be able to answer any of the questions. You’ll be sacrificingsome easy points if you do.

LSAT PREP _____________________________________________________________ LSAT Test XVII Explained: Section IV

© K A P L A N 83

The Questions:

22. (B)This choice encompasses the passage’s topic (African drought); scope (a widespreadhypothesis about the cause of the drought); and purpose (to describe and take issue withthis hypothesis).

(A) Au contraire. The author says that the “cooling hypothesis,” which is based on the roleof atmospheric dust, is “not well supported” by the meteorological facts.(C) focuses on a detail in ¶2, not the passage as a whole.

(D) Like (C), this choice focuses on a detail in ¶2. Besides, this view is held by proponentsof the “cooling hypothesis.” It’s not the author’s view.

(E) is beyond the scope of the passage, which offers no information about when the Africandrought is likely to end.

• The two most frequent “trap” choices on global questions are choices that focus ondetails, and choices that go beyond the scope of the text. Watch out for them.

23. (B)In ¶3, the author says that the “cooling hypothesis” is “not well supported”; and he offers anumber of meteorological facts that appear to undermine it. But he knocks the hypothesisin a very restrained tone. That’s why “cautious skepticism” is a better description of hisattitude than “vehement opposition” (A).

(C) The author’s not “ambivalent” about the hypothesis. Lines 48-50 suggest that he gave itsome credence at some point, but that's only the slightest of suggestions.

(D) and (E) suggest that the author’s a proponent of the hypothesis. That clearly isn’t thecase.

• When you’re asked about the author’s attitude, you’ll generally be able to eliminatethree choices with relative ease. Of the last two, the more moderate one will usually(but not always) be correct because most LSAT passages, if they are opinionated,contain moderate opinions appropriate to scholarly discourse. And should someauthor's opinions be vehement, the evidence for that vehemence will be clearlyplaced in the passage for all to see.

LSAT PREP _____________________________________________________________ LSAT Test XVII Explained: Section IV

84 © K A P L A N

24. (D)Lines 33-34 is where this “circumpolar vortex,” so called, is first mentioned; and what “iscausing” (line 32) it? Lines 30-32 tell us. Expansion of the circumpolar vortex, according tothose who endorse the “cooling hypothesis,” occurs when there’s a growth in thetemperature differential between the tropical latitudes and more northerly latitudes.

(A) focuses on the tropics exclusively—no can do.

(B) distorts a detail in ¶2. If anything, the circumpolar vortex might expand if there’sheavier than normal snowfall for an extended period.

(C) is beyond the scope of the passage, which doesn’t mention temperature differentialsbetween water and land.

(E) The significant difference is between the tropical latitudes, on the one hand, and themiddle and high latitudes, on the other.

• When students complain about “detail questions,” this and Q. 25 are theirnightmares. Both are prime examples of why it’s important to go back to the passagefor details. All of Q. 24's choices might have looked convincing at first glance, andonly by rereading lines 29-34 could you confidently zero in on choice (D).

25. (A)Note that the question asks specifically about Northern Hemisphere landmasses, not aboutthe Northern Hemisphere in general. That makes the information in lines 8-12 relevant here,and clearly those temperatures declined after 1945. (A) just describes that the other wayround: Temps were higher, before. Incidentally, information in lines 58-63 isn’t relevantbecause it refers to the Northern Hemisphere in general.

(B) Since we’re not given actual figures, we have no way to compare the pre-1945 period tothe 1980s. All we know is that temperatures were warmer in both eras than they werebetween 1945 and the early 1970s.

(C) Lines 8-12 state otherwise.

(D) and (E) We aren’t given the actual data that would permit either comparison.

• This question, like Q. 24, is very difficult; it may be arguable which is the hardest inthe entire Reading Comprehension section, but it's surely one of these two. Thus,either or both are good candidates to tackle after you’ve completed all of the otherquestions (unless, of course, you happen to see an answer right away). Remember,questions like these one aren’t worth more than easy questions; so, don’t waste a lotof time on them if you don’t see a quick path to the answer.

LSAT PREP _____________________________________________________________ LSAT Test XVII Explained: Section IV

© K A P L A N 85

26. (D)¶1 sets out the “cooling hypothesis,” which addresses the cause of the African drought; therest of ¶1—as well as all of ¶2—describes the hypothesis and its supporting evidence; and¶3 disputes the hypothesis.

(A) An opposing point of view—the author’s—isn’t presented until the end of the passage.The formal hypothesis comes first.

(B) The author’s view and that of proponents of the “cooling hypothesis” aren’t reconciled;they’re simply discussed.

(C) The author questions the “cooling hypothesis,” but he never “amends” it. As far as weknow, he simply rejects it.

(E) What second theory? The author doesn’t offer one of his own.

• If you’re asked about the “organization” of the passage, think about its structurebefore you browse among the choices. This type of question lends itself well to pre-phrasing.

27. (E)Lines 36-40 indicate that those who endorse the “cooling hypothesis” believe that the sub-Saharan drought has been caused by a shift in normal wind patterns. This shift, theycontend, has kept monsoon rains from reaching the sub-Saharan region. Thus, they wouldprobably say that the return of such rains means that wind patterns have returned tonormal.

(A) Au contraire. According to supporters of the “cooling hypothesis,” an increase in iceand snow coverage would be likely to keep the rains at bay.

(B) Au contraire aussi. Again, according to hypothesis advocates, such a developmentwould likely perpetuate the sub-Saharan drought.

(C) Hypothesis advocates never directly link conditions in the tropics with sub-Saharanrain.

(D) Another au contraire choice. Lines 36-40 identify this development as part of the causalchain of sub-Saharan drought.

• Questions that test relationships will often contain au contraire choices. The bestapproach to questions like this one is to go back to the passage, reread therelationship, and then pre-phrase an answer. When a choice matches your pre-phrased answer, you can be confident that it’s correct.

I.N. LL3118 Rev.1 Printed in the USA